9/4-MBE CONSTITUTIONAL LAW

अब Quizwiz के साथ अपने होमवर्क और परीक्षाओं को एस करें!

Question 7135 State A derived much of its business from tourism based upon privately owned cruise lines incorporated in State A. State B, which is located just north of State A, recently enacted a statute creating a yearly licensing requirement for any cruise lines not incorporated in State B that use State B ports. The licensing process was costly and the licenses were difficult to obtain. There was no similar licensing requirement for State B cruise lines using the ports in State B. Congress has not enacted legislation regarding the regulation of cruise lines using out-of-state ports. The cruise lines in State A typically travel north and must use the State B ports in order to maintain the facilities and services offered by their cruise lines. If a cruise line located in State A challenges the constitutionality of the State B licensing requirement, which of the following would provide the strongest argument against the requirement? Answers: The Privileges and Immunities Clause of Article IV The Equal Protection Clause The Due Process Clause The Dormant Commerce Clause

Answer choice D is correct. The Dormant Commerce Clause limits the power of states to legislate in ways that impact interstate commerce. If Congress has not enacted legislation in a particular area of interstate commerce, the states are free to regulate so long as the state or local statutes do not (i) discriminate against out-of-state commerce, (ii) unduly burden interstate commerce, or (iii) regulate extraterritorial (wholly out-of-state) activity. Here, Congress has not enacted legislation regarding the regulation of out-of-state cruise lines using ports in other states. Thus, State B was free to regulate it as long as it did not discriminate against out-of-state commerce or unduly burden it. However, the State B statute, on its face and in practice, is discriminatory because it protects local economic interests at the expense of out-of-state competitors. If the cruise line challenges the constitutionality of the statute based on the Dormant Commerce Clause, then State B must establish that (i) an important local interest is being served, and (ii) no other non-discriminatory means are available to achieve that purpose. State B is protecting cruise lines incorporated in State B at the expense of State A cruise lines, but there is no apparent local interest being served, and other non-discriminatory means would be available to serve a local interest, such as not overcrowding the use of ports in State B. For this reason, answer choice D is the best answer. Answer choice A is incorrect because the Privileges and Immunities Clause is not implicated here. Article IV, Section 2, known as the Comity Clause, provides that "the citizens of each state shall be entitled to all privileges and immunities of citizens in the several states." The Comity Clause, in essence, prohibits one state from discriminating against the citizens of another state. In this context, the term "citizen" does not include corporations. Here, the Comity Clause does not apply because the licensing requirement is only applicable to corporations. Answer choice B is incorrect because this question does not indicate any equal protection violation, as there is neither a suspect class nor a fundamental right at issue. Even though the right to travel is a fundamental right, that right is not necessarily implicated by a licensing requirement for the use of a port. Answer choice C is incorrect because the question indicates a commerce regulation, which is the central issue here, rather than a due process issue.

Question 1387 A professional football player was accused of sexually assaulting a young man in a local bar. The prosecution refused to divulge the name of the alleged victim. Nevertheless, a reporter determined the alleged victim's identity through interviews of eyewitnesses present at the scene. The reporter's newspaper published the alleged victim's name, causing the alleged victim extreme embarrassment. The alleged victim sued the newspaper and the reporter. Can the alleged victim recover damages? Answers: Yes, the alleged victim can recover from the reporter. Yes, the alleged victim can recover from the newspaper. Yes, the alleged victim can recover from the newspaper and the reporter. No, the alleged victim cannot recover from either the newspaper or the reporter.

Answer choice D is correct. The First Amendment shields the media from liability for publication of a lawfully obtained private fact, e.g., the identity of a rape victim, so long as the news story involves a matter of public concern. Therefore, neither the newspaper nor the reporter are liable for legally gathering and publishing these private facts concerning a matter of public concern. Answer choice A is incorrect because the alleged victim cannot recover here, as the reporter lawfully obtained the information in a story involving a matter of public concern. Answer choice B is incorrect because although the press has no greater First Amendment rights than does the general public (i.e., there is no special privilege allowing the press to invade the rights of others), the First Amendment shields the media from liability for publication of a lawfully obtained private fact so long as the news story involves a matter of public concern. Answer choice C is incorrect because both the newspaper and the reporter are considered media protected from liability in this situation. (Note: The First Amendment also shields the media from liability for publishing information that was obtained illegally by a third party as long as the information involves a matter of public concern, and the publisher neither obtained it unlawfully nor knows who did.)

Question 6601 The State Department adopted a rule that passports issued to U.S. citizens who were born in disputed territory would list their city of birth rather than country of birth when the President had not recognized the disputed territory as belonging to a specific country. Congress passed legislation that, among other things, required the State Department to list the country indicated by passport applicants as their place of birth on their issued passport. Although the President signed the bill, he noted that this particular requirement unconstitutionally invaded his power over foreign affairs. Shortly after his birth in a disputed territory to parents who were United States citizens, a passport applicant requested through his parents that he be issued a passport that listed a specific country as his place of birth. The passport office refused. The passport applicant filed suit in federal court seeking injunctive relief that mandates the passport office to issue the applicant a passport that lists a country as his place of birth. How should the court respond? Answers: Deny the injunction, because the President has exclusive power over the recognition of a foreign country. Grant the injunction, because Congress has the power to override a presidential decision regarding the recognition of a foreign country. The court should refuse to hear the matter as a nonjusticiable political question. The court should refuse to hear the matter because the passport applicant lacks standing.

Answer choice A is correct. The President represents and acts for the United States in day-to-day international affairs. In addition to appointing and receiving ambassadors, the President has the exclusive power to recognize a foreign government. Consequently, because the President has exclusive power over the recognition of a foreign country, the court should not grant the injunction because it would require the passport office to recognize a country that the President has not yet recognized. Answer choice B is incorrect because Congress lacks the power to reverse a presidential decision with regard to the recognition of a foreign country. Answer choice C is incorrect because a political question arises when the Constitution has assigned decision making on this subject to a different branch of the government, or the matter is inherently not one that the judiciary can decide. The judiciary is not being asked to recognize a country, a decision that would be assigned to the President. Instead, the judiciary is being asked to interpret applicable law, determine whether it can be enforced, and then determine whether injunctive relief is appropriate. Because this determination is well within judicial competence, the court need not refuse to hear the matter as a political question. Answer choice D is incorrect. Although the passport applicant was himself too young to appreciate the significance of the matter and did not suffer any psychological injury as a consequence, he suffered an injury to a congressionally created right to have his passport reflect a particular country of birth. Therefore, the plaintiff has standing.

Question 7139 Due to a rising concern for children with undisclosed food allergies, as well as preserving a controlled environment in school cafeterias, a city enacted an ordinance stating that only lunches made without a specific list of ingredients commonly causing food allergies could be prepared and served in school cafeterias. In addition, to avoid any lunches that could cause food allergies, the ordinance stated that only students who ordered lunch in the cafeteria could eat there; students who brought their lunches from home were not permitted inside of the cafeteria during lunch time. Instead, these students were required to eat their lunches in a different room. A student who only ate certain foods in accordance with his religious beliefs brought his lunch from home. The lunch did not contain any of the prohibited ingredients listed in the ordinance. All of the student's friends ate their lunch in the cafeteria, but the student was forced to eat his lunches in another room. If the student's parents challenge the constitutionality of the ordinance under the Equal Protection Clause of the Fourteenth Amendment, are they likely to succeed? Answers: Yes, because the ordinance is not narrowly tailored to address the dangers posed by food allergies. Yes, because the ordinance has a disparate impact on children with religious dietary restrictions. No, because the ordinance is rationally related to a legitimate governmental interest. No, because the ordinance applies to all students.

Answer choice C is correct. The Equal Protection Clause of the Fourteenth Amendment provides that states are generally prohibited from passing legislation that treats similarly situated persons differently. To trigger strict or intermediate scrutiny, there must be discriminatory intent on the part of the government or a classification based on a suspect or quasi-suspect class. Strict scrutiny also applies if a fundamental right is involved. The rational basis standard is used in all cases in which one of the higher standards does not apply. A law passes the rational basis standard of review if it is rationally related to a legitimate governmental interest. This is a test of minimal scrutiny. Here, students who bring their lunches from home are being treated differently from students who eat in the cafeteria because they must eat in a separate room. However, the ordinance has a rational basis that is related to the legitimate government interest of preventing complications due to food allergies. Because this ordinance does not target a suspect class or involve a fundamental right and has no discriminatory intent, it passes rational basis review. Therefore, the parents' challenge is likely to fail. Answer choice A is incorrect because this ordinance only needs to meet rational basis scrutiny, and need not be narrowly tailored to its purpose. Answer choice B is incorrect because the fact that legislation has a disparate effect on people of different races, genders, or religions, without discriminatory intent, is insufficient to trigger strict or intermediate scrutiny. Answer choice D is incorrect. The fact that the ordinance applies to all students does not mean it is constitutional on that basis because it still differentiates between students who bring their lunches from home and those who buy their lunches in the cafeteria, requiring the former to eat lunch in a separate room.

Question 6277 A state law authorized slot machines at various locations in the state and imposed a tax on revenues from the slot machines at a rate of 35 percent. In response to the financial peril in which the state riverboat industry found itself, the state lowered the tax rate on revenues from slot machines found on riverboats in the state, which represented a substantial portion of the riverboat industry's income, to 20 percent. The owners of the venues paying the 35 percent tax rate on their slot machine revenue filed an action in federal court challenging the constitutionality of the different tax rates depending upon the type of venue because it violates the Equal Protection Clause. Is the court likely to rule in favor of the plaintiffs? Answers: Yes, because the state was favoring one intrastate venue over other types of venues. Yes, because the law violated the geographic uniformity requirement imposed by Article I, Section 8. No, because the subsidy exception of the Dormant Commerce Clause applies. No, because the different tax rates were rationally related to a legitimate state interest.

Answer choice D is correct. Since the plaintiffs have challenged the differential tax rates based on equal protection grounds, and because the plaintiffs are not members of a suspect or quasi-suspect class, the discriminatory tax rate is only subject to the rational basis test. Here, the state acted to aid a state industry (the riverboat industry) that was in financial peril (a legitimate state interest) by lowering the tax rate on one form of revenue that generated a substantial portion of its income. While the state could have undertaken other actions to aid the riverboat industry, the modification of the tax rate on slot machine revenue was rationally related to the state's interest in reducing the riverboat industry's financial peril. Consequently, the court is likely to rule against the venue owners paying the 35 percent tax rate. Answer choice A is incorrect because, unlike state action that discriminates against interstate commerce, there is no specific constitutional restriction on state action that discriminates against intrastate business in favor of another intrastate business. Moreover, the plaintiffs' challenge is not based on the Dormant Commerce Clause, but on the Equal Protection Clause. Answer choice B is incorrect because the geographic uniformity requirement is a constitutional restriction imposed on the federal government, not on the states. Moreover, as stated above, the plaintiffs' challenge is based on the Equal Protection Clause. Answer choice C is incorrect because, while a state may provide a subsidy for its own citizens under the Dormant Commerce Clause (e.g., a lower tuition rate for in-state students attending college), the plaintiffs' challenge is based on the Equal Protection Clause.

Question 3263 A county government enacted an ordinance regarding the processing of solid waste within its boundaries. The county required all solid waste to be brought to a local transfer station that was owned and operated by a private company. Anyone who brought solid waste to the transfer station was charged a "tipping fee." The county also required each commercial trash hauler to purchase a permit in order to collect solid waste in the county. Although the solid waste itself did not have an economic value, the combined cost of the "tipping fee" and the permit was fixed at an amount to ensure that the private company's operation of the transfer station was profitable. Prior to the enactment of the ordinance, a local trash hauler collected solid waste in the county and delivered it to an out-of-state facility, which charged much less than the tipping fee. The local trash hauler filed an action challenging the constitutionality of the county ordinance. The court ruled in favor of the local trash hauler. Is the court's ruling correct? Answers: Yes, because the ordinance discriminates against interstate commerce. Yes, because a local government may not indirectly regulate interstate commerce. No, because trash collection is a traditional function of local governments. No, because the solid waste did not have economic value.

Answer choice A is correct. A state or local government generally cannot mandate an action that discriminates against interstate commerce. Here, the county ordinance requires the use of a local waste transfer facility instead of the out-of-state facility that the plaintiff had been using before the passage of the ordinance. Because the ordinance protects a local interest at the expense of an out-of-state competitor, it is discriminatory. Accordingly, it is impermissible under the Dormant Commerce Clause. Answer choice B is incorrect because the Dormant Commerce Clause does not prohibit a local government from indirectly regulating interstate commerce. The clause only requires that such regulation not discriminate against out-of-state commerce, unduly burden interstate commerce, or deal only with extraterritorial activity. Answer choice C is incorrect because, although there is an exception to the Dormant Commerce Clause for an activity such a trash collection that is a traditional function of local governments, this exception applies only when the activity is conducted by the local government. Here, the transfer station was owned and operated by a private company. Answer choice D is incorrect because, although the solid waste did not have economic value, it was an item that moved in interstate commerce. Consequently, the Dormant Commerce Clause governed the local government's regulation of it.

Question 3265 Under a state's general consumer statute, deceptive acts in the conduct of trade or commerce are unlawful. A smoker of cigarettes that were labeled "light" alleged that the cigarettes did not contain lower levels of tar and nicotine than non-light cigarettes, and sued the manufacturer for violation of the state statute. Federal law requires specific labeling of cigarette packages, and prohibits states from requiring additional statements relating to smoking and health on cigarette packages. Federal law also prohibits a state from imposing requirements on the advertising or promotion of a manufacturer's cigarettes. The stated purpose of the federal law is to warn the public of the health risks of smoking while protecting the economy from the ill effects of non-uniform requirements. The manufacturer contends that federal preemption applies to prohibit the smoker's claim based on the state statute. Both sides agree that the cigarette manufacturer has complied with the federal labeling laws. Which of the following is not a viable argument that the smoker may make against the manufacturer's preemption defense? Answers: Federal preemption must be expressly stated. A federal law is narrowly read when determining whether it preempts a state statute. The state consumer statute is aimed at consumer protection rather than public health. Neither purpose of the federal law would be served by limiting the state's authority to prohibit deceptive statements in cigarette advertising.

Answer choice A is correct. Federal preemption of a state statute may be implied as well as expressly stated. Consequently, the argument that federal preemption must be expressly stated is invalid. Answer choice B is incorrect because preemption is disfavored, and thus a federal law is narrowly read to avoid preemption when possible. Answer choice C is incorrect because the federal law preempts a requirement based on smoking and health, and the state statute is a general consumer protection that imposes a duty not to deceive. This restriction is not based on smoking and health, and thus may not be preempted by the federal law. Answer choice D is incorrect because the smoker has a viable argument that the state statute does not conflict with the purposes of the federal law. Under these circumstances, a court could find that federal law does not impliedly conflict with the state statute.

Question 7058 Compared to traditional incandescent light bulbs, energy-efficient light bulbs typically use approximately 25%-80% less energy. Although most manufacturers of light bulbs have switched to making energy-efficient light bulbs, manufacturers of light bulbs in 10 states still only make traditional incandescent light bulbs. In order to discourage the use of traditional incandescent bulbs, which accelerates the negative consequences of climate change and results in more waste in landfills, Congress imposed a tax on the sale of traditional light bulbs by any manufacturer. The stated purpose of the tax is to generate revenue to help further research to prevent climate change. Is this tax constitutional? Answers: Yes, because there is geographical uniformity in the application of the tax. Yes, because the tax is necessary and proper to further a compelling governmental interest. No, because the tax exceeds the point at which financial pressure turns into unconstitutional compulsion No, because this direct tax will not be apportioned evenly among the states.

Answer choice A is correct. The requirement that indirect federal taxes (i.e., duties, import and excise taxes) must be uniform throughout the United States has been interpreted to mean geographical uniformity only; the product or activity at issue must be identically taxed in every state in which it is found. Here, the traditional light bulbs are being identically taxed, even if the only manufacturers of them are in ten states. Answer choice B is incorrect because the government has no burden to prove that a tax is necessary to any compelling governmental interest. Instead, the General Welfare Clause has been interpreted as permitting Congress to exercise its power to tax for any public purpose. Answer choice C is incorrect because this limitation does not apply to coercion of private actors. Answer choice D is incorrect because sales taxes are indirect taxes, and are not subject to an apportionment requirement.

Question 1383 A student joined a small national organization during her freshman year of college after several of her friends, who were active in the organization, told her about the organization's annual all-expense paid ski trip for its card-carrying members. The student attended a recruitment drive, signed a pledge of loyalty to the party, paid her annual dues, and received an organization pin. The student never attended a meeting or read the organization's literature she was given. The student later joined other members of the organization at the ski resort. During a meeting around the ski lodge fireplace, the student learned for the first time that the organization was a radical organization. The organization's members were preparing to use subversive means to achieve their objective of installing the organization's spiritual leader as Supreme Dictator of the United States. To this end, the organization was stockpiling mind control serum and planned to poison the nation's water supply. After returning home, the student consciously avoided members of the organization and never participated in the organization's activities again, but the student's name remained on the organization's active member roster. Three years later, the student neared graduation and applied for federal employment. Can the student be denied public employment based upon her membership in this organization? Answers: No, because the student did not personally participate in the organization's subversive plot. No, because the student did not intend to install the organization's spiritual leader as Supreme Dictator. Yes, because the student continues to be listed as an active member of a subversive organization. Yes, because the student has knowledge of the organization's illegal objectives and failed to report them to the authorities.

Answer choice B is correct because the student lacked the specific intent to further the organization's illegal objectives; this intent is necessary in order to punish her based upon her association with the group. The First Amendment's freedom of association protects the right to form or participate in any group, club, or other organization virtually without restriction, but the right is not absolute. The government may justifiably infringe the right of association to advance a compelling state interest, such as preventing discrimination or subversive activities designed to undermine the democratic process. A person may only be punished or deprived of public employment based on their political affiliation if that individual (i) is an active member of a subversive organization, (ii) has knowledge of the organization's illegal activities, and (iii) has a specific intent to further those illegal objectives. Answer choice A is incorrect because a person does not have to personally or physically participate in a subversive organization's overt activities to be punished if the person is a member of that organization, aware of the organization's illicit goals, and intends that those subversive objectives be furthered. Answer choice C is incorrect because her inclusion on the active member list is insufficient for purposes of denying her public employment. Answer choice D is incorrect because an individual cannot be denied public employment based simply upon knowledge of the organization's objectives. She would also need the intent to further those objectives.

Question 7416 An attorney brought an action challenging the constitutionality of a recently enacted federal statute that permits the seizure of confidential communications between a U.S. citizen located in the United States and a foreign national living abroad. The attorney, a U.S. citizen, represented foreign nationals who were held in federal custody, and although she was unable to establish that a particular conversation with one of her clients had been intercepted, she contended that the statute had a chilling effect on her First Amendment freedom of speech rights. The attorney is seeking an injunction against the appropriate federal officials to prevent implementation of the statute. Who bears the burden of establishing that the attorney has standing to bring this action? Answers: The attorney, because she cannot establish a present injury. The attorney, because she is the plaintiff. The federal officials, because the attorney is asserting that the statute violates her constitutional rights. The federal officials, because the injunction sought by the attorney would prevent her injury.

Answer choice B is correct. A federal court cannot decide a case unless the plaintiff has standing to bring it. To have standing, a plaintiff bears the burden of establishing three elements—(i) injury in fact, (ii) causation, and (iii) redressability. Answer choice A is incorrect because, while a plaintiff must establish an actual or imminent injury in order to have standing, having a present injury does not shift the burden of establishing standing to the defendant, as the answer choice implies. Regardless of whether the plaintiff's injury is present or merely imminent, the plaintiff bears the burden of establishing standing. Answer choice C is incorrect. Although the government may bear the burden of establishing that the strict scrutiny test has been satisfied if a statute infringes on a constitutional right, the plaintiff who asserts that a statute does so must first establish that she has standing to challenge the statute. Answer choice D is incorrect. Although a plaintiff must establish that the relief requested will prevent or redress the injury in order to have standing, redressability is but one of the elements that the plaintiff must establish in order to have standing. The existence of redressability does not shift the burden of establishing standing to the defendants.

Question 7392 In response to growing concerns about the overcrowding of landfills with scrap metal, Congress passed a statute requiring all unwanted vehicles to be disposed of at federally licensed auto-recycling facilities. These facilities were able to recycle more components of vehicles than most other recycling facilities. However, due to the high operating costs of these facilities, the cost of disposing the vehicles was much higher than the cost of disposing them at general purpose recycling facilities. A state wants to dispose of its fleet of decommissioned trucks at a state-operated recycling facility. However, this facility is not federally licensed. Is the state permitted to dispose of its decommissioned trucks at the state-operated facility? Answers: No, because the federal statute was passed pursuant to Congress's power to legislate for the general welfare. No, because the federal statute regulates interstate commerce. Yes, because the market-participant exception applies. Yes, because there is a presumption against preemption in an area governed by the state's police power.

Answer choice B is correct. Congress's power under the Commerce Clause extends to environmental laws such as this auto-recycling statute. As such, Congress can invoke the Necessary and Proper Clause to preempt all conflicting state laws, and the federal statute governs by virtue of the Supremacy Clause. Answer choice A is incorrect. Although Congress is permitted to tax and spend for the general welfare, it does not have the power to legislate for the general welfare. Thus, this is not a proper basis for the federal statute and would not support its enforcement against the state. Answer choice C is incorrect. The market-participant exception allows a state to behave in a discriminatory fashion against nonresident commerce if it is acting as a market participant. It does not permit a state to violate federal law regarding commerce. Answer choice D is incorrect because, although the state wants to dispose of its trucks, this disposal likely does not fall under the state's police powers. Therefore, the federal statute will govern.

Question 4183 A state passed a law prohibiting the construction of any structure within 100 feet of the shoreline of an inland lake. A landowner owned a piece of empty lakefront property that extended 200 feet beyond the shoreline. The landowner sued the state, alleging that the law constituted a taking. Which of the following, if raised at trial, would not factor into the court's determination of whether or not a taking has occurred? Answers: The degree to which the law benefits society at large. Whether the law substantially advances a legitimate government interest. The owner's intentions in possessing the property. The owner's ability to sell the property to a third party.

Answer choice B is correct. Generally, a governmental regulation that adversely affects a person's property interest is not a taking, but when a regulation so substantially hinders a person's property interest as to make it virtually valueless, that law may be considered a "regulatory taking." In determining whether a regulation constitutes a taking, the court will consider: (1) the economic impact of the regulation on the property owner; (2) the extent to which the regulation interferes with the owner's reasonable, investment-backed expectations regarding his use of the property; and (3) the character of the regulation, including the degree to which it will benefit society, how the regulation distributes the burdens and benefits among property owners, and whether the regulation violates any of the owner's essential attributes of property ownership, such as the right to exclude others from the property. A court generally will not examine whether a regulation substantially advances a legitimate governmental interest. Here, all the factors are valid considerations for the court except whether or not the law advances a legitimate government interest. Answer choice A is incorrect because a court may consider the degree to which a law will benefit society. Answer choice C is incorrect because the court may consider the extent to which the law interferes with the owner's reasonable expectations regarding the use of his property; if the owner had purchased the property with no intent to build a structure on it, his claim would be significantly weakened. Answer choice D is incorrect because the ability to resell the property speaks to the economic impact of the property regulation to the owner. It may also be relevant in determining whether any one of the essential attributes inherent in property ownership has been violated.

Question 4367 A state enacted a law prohibiting anyone convicted of a felony from voting in state elections. A group of released felons filed suit in federal court, alleging that the statute violated their constitutional rights. Is the court likely to uphold this statute as constitutional? Answers: No, because it infringes upon the fundamental right to vote. No, because it violates the Equal Protection Clause. Yes, as permitted by Section 2 of the Fourteenth Amendment. Yes, pursuant to the Privileges and Immunities Clause.

Answer choice C is correct. As permitted by Section 2 of the Fourteenth Amendment, a state may prohibit a felon from voting, even one who has unconditionally been released from prison. Answer choice A is incorrect because while voting is typically considered a fundamental right and would therefore have to meet the burden of strict scrutiny, a state may prohibit a felon from voting. Answer choice B is incorrect because felons are not a protected class; accordingly, only the rational basis test would apply. Answer choice D is incorrect because the Privileges and Immunities Clause prohibits discrimination against out-of-state citizens and does not apply to these facts.

Question 3231 Condominiums located within a particular city block were known to be the center of illegal narcotic sale operations. Many individuals living within that city block had been arrested multiple times in connection with the illegal sales, and neighbors were outraged that the local police department did not attempt to do something more severe than individual, sporadic arrests. After substantial neighborhood protests, the city took action by condemning the entire city block, thereby seizing all condominiums within the block. The city also towed all cars known to be used in connection with transporting narcotics outside the city center. A man whose home and car were both seized in connection with the alleged crimes filed suit, alleging that he was never notified of the city's intent to seize his home and vehicle. Are the seizures of the home and the vehicle proper in light of the principles of procedural due process? Answers: The seizure of the home and vehicle are both improper. The seizure of the home is proper, but the seizure of the vehicle is improper. The seizure of the vehicle is proper, but the seizure of the home is improper. The seizure of neither the home nor the vehicle is improper.

Answer choice C is correct. Generally, the government is required to provide the owner of real property with notice and a hearing prior to seizure of the property pursuant to a forfeiture statute. However, the government does not need to provide notice prior to the seizure of personal property. Here, the city provided no notice or hearing for either the home or the vehicle. The city was required only to provide notice and a hearing for the seizure of the home but did not do so. Accordingly, the seizure of the home is improper though the seizure of the vehicle is not. Answer choice A is incorrect because the seizure of the vehicle was proper, as no notice or hearing is required prior to the seizure of personal property in a forfeiture situation. Answer choice B is incorrect because the seizure of the home is improper in light of the lack of a notice and hearing. Answer choice D is incorrect because the seizure of the home was improper due to the failure to provide a notice and hearing prior to the seizure.

Question 3219 A violent storm results in severe property damage and extreme loss of life in States A, B, and C, which all become the subjects of emergency declarations by the President. The National Guard is present for three months in each state while recovery occurs. A year later, most of the damaged property has been reconstructed, and no current threat to life or property remains. During a time of year when similar storms are common, the President instructs the National Guard to set up a presence within various points in all three states. Congress had previously enacted legislation allowing the President to instruct the National Guard in such a way. The President neither requested nor received authority from the governor of State C, the smallest state of the three, but did receive authority from the governors of States A and B. Is the President's action constitutional? Answers: No, because the governor of State C did not authorize the presence of the National Guard. No, because an emergency situation no longer existed at the time of the President's order. Yes, because the President's actions are aligned with permissible Congressional authority. Yes, because the President sent the National Guard to all three states.

Answer choice C is correct. National Guard units are under the dual control of the federal and state governments. Under the Militia Clauses (Art. I, Sec. 8, Cl. 15, 16), Congress has the power to authorize the President to call National Guard units to execute federal laws, suppress insurrections, and repel invasions. This constitutional authority extends to use of National Guard units in domestic situations and non-emergency circumstances, and is not subject the approval or veto of the governor of a state. Answer choice A is incorrect because the President retains the authority to order units of the National Guard into service within a state even without express consent from the state's governor. Answer choice B is incorrect because the President can authorize the National Guard even when no emergency exists. Answer choice D is incorrect because it is irrelevant. There is no requirement that the President order units of the National Guard into service to every state suffering from an emergency.

Question 6288 A police officer made a sexually explicit video that the officer marketed on the internet. The video, which was made while the officer was off-duty, correctly identified the maker of the video as a police officer for the city in which the officer lived. The video depicted the officer engaged in indecent acts while wearing a police officer's uniform and brandishing a badge and gun. The officer had purchased the uniform, badge and gun from a Halloween costume shop; nothing in the video was acquired from the police department. When the chief of police learned of the video, she requested that the officer remove the video from the internet because it violated the police department's policy against immoral and unbecoming conduct. The officer, asserting that the video was intended as a parody of police behavior, refused to remove the video from the internet and was fired as a consequence. The officer brought an action against the police chief, seeking reinstatement to the police force. The officer contended that he had been dismissed for exercising his free speech rights. How should the court rule on the officer's action? Answers: For the officer, because the intent of the video was to be a parody of police behavior. For the officer, because the officer truthfully indicated his profession and his employer. For the police chief, because the officer's video did not constitute speech on a matter of public concern. For the police chief, because a government employee does not enjoy free speech rights with respect to his employment.

Answer choice C is correct. When a government employee contends that his rights under the Free Speech Clause of the First Amendment, as made applicable to state and local government action through the Fourteenth Amendment, have been violated by his employer, the employee must show that he was speaking as a citizen on a matter of public concern. Even when an employee is speaking as a citizen on a matter of public concern, the First Amendment interest of the employee must be balanced against the interest of the state, as an employer, in effective and efficient management of its internal affairs. Here, although police behavior is a matter of public concern, there is no indication that this sexually explicit video actually addresses that matter of public concern in any meaningful way. Even if the sexually explicit video does have some tangential relation to the public concern of police conduct, the police department's interests in promoting the efficiency of the public services it performs through its employees likely prevails over the officer's interests in profiting from a sexually explicit video. Answer choice A is incorrect because the content of the speech, not its intent, determines its level of protection, and the officer has not adequately established that this sexually explicit video actually addresses that matter of public concern in any meaningful way. Answer choice B is incorrect because, while a government employee's false or misleading speech may justify suppression of the speech by the government employer, the mere fact that the speech is neither false nor misleading does not result in it enjoying First Amendment protection. Answer choice D is incorrect because, although a government employee's speech may enjoy protection under the First Amendment from retaliation by the government employer, in order to enjoy such protection, the speech must be about a matter of public concern, and the officer has failed to establish that this sexually explicit video constitutes speech regarding police behavior.

Question 5942 A state highway administration, acting pursuant to statutory authorization by the state legislature, has promulgated rules for large electronic billboards located along roads maintained by the state. These rules are concerned with the potential for driver distraction and the ensuing adverse consequences for highway safety. Among the rules is one that bans the graphic display of violence. The producer of a movie wants to promote the movie through a short clip from the movie on billboards subject to this rule. The clip contains a graphic display of violence. The producer has filed an action in the appropriate federal court challenging the state highway administration's rule as a violation of the First Amendment as applicable to the states through the Fourteenth Amendment. Of the following standards, by which should the state highway administration's rule be judged? Answers: As a rule that deals with a matter traditionally subject to regulation, it must be upheld unless it is arbitrary or irrational. As a regulation of commercial speech, there must be a reasonable fit between the government's ends and the means chosen to accomplish those ends. As a time, place, or manner restriction, it must be narrowly tailored to serve a significant governmental interest. As a content regulation, it must be necessary to achieve a compelling governmental interest.

Answer choice D is correct. Although viewpoint neutral, the state highway administration's rule is a content-based regulation of speech because it prohibits the graphic display of violence. As such, the strict scrutiny standard of review applies. The rule must be necessary to achieve a compelling governmental interest. Answer choice A is incorrect. Although content regulation of speech that falls into certain historical categories, such as obscenity or defamation, may be permitted, the depiction of violence is not one of those categories. Answer choice B is incorrect. Although the producer is concerned with the impact of the rule on his commercial speech, the rule is not limited to commercial speech. Instead, the rule applies to any display that appears on the billboard, even those that have a noncommercial purpose, such as public service announcements. Answer choice C is incorrect. Time, place, or manner restrictions must be narrowly tailored to serve a significant governmental interest, and arguably, rules regarding electronic billboards specifically are time, place, and manner restrictions. However, the portion of the rule affecting the movie producer is a restriction of the content of expression. The more stringent standard applies.

Question 6297 The dean of the architecture department at a state university informed a graduate-level architecture student that if she did not improve her academic performance, she would not be allowed to remain a student at the university. When the student's grades did not improve the following semester, a faculty committee recommended that she be dismissed after careful review of her academic record. The student was not allowed to present evidence to the committee or attend the committee meeting. The dean accepted the committee's recommendation and subsequently dismissed the student. The student filed a constitutional challenge to her dismissal in federal court, contending that she had been deprived of her procedural due process rights. The court determined that she had been deprived of a liberty interest, but not a property interest, by being dismissed as a student. Is the court likely to rule in favor of the student? Answers: No, because the court determined that she had not been deprived of a property interest due to her dismissal. No, because she was not entitled to a hearing since she was dismissed for academic reasons. Yes, because the court determined that she had been deprived of a liberty interest by her dismissal. Yes, because she was denied the right to present evidence to the committee and to attend the committee meeting.

Answer choice B is correct. A student is not entitled to a hearing with regard to dismissal for academic reasons from a public institution of higher learning. Since the architecture student was provided with notice of her academic shortcomings and an opportunity to correct those shortcomings, she was provided with adequate procedural due process. Answer choice A is incorrect because a person who has been deprived of a liberty interest need not also be deprived of a property interest in order to be entitled to procedural due process. Answer choice C is incorrect because, even though the court determined that the student had been deprived of a liberty interest, she was not automatically entitled to a hearing. Answer choice D is incorrect. The student did not have a right to a hearing with regard to her academic dismissal, so she did not have the right to present evidence to the committee or attend the committee meeting.

Question 4177 A state legislature passed a law requiring employers to provide their employees with health insurance that covered certain prescription drugs. Violation of this statute was considered a crime that subjected the offender to fines, which were described in detail in the statute. The law was effective immediately. An employer did not provide its employees with insurance that covered the required drugs, and argued that such drugs were prohibited by the religion practiced by the employer. The employer filed a complaint in federal court asserting that the law was unconstitutional and asked for a preliminary injunction against the attorney general to prevent him from enforcing the statute while the case was heard. The attorney general filed a motion to dismiss, asserting that the federal court did not have jurisdiction to hear the case. How should the federal court rule on the attorney general's motion to dismiss? Answers: Grant the motion based on prudential grounds. Grant the motion based on the doctrine of abstention. Deny the motion, because the employer has taxpayer standing due to the imposition of fines. Deny the motion, because the employer's injury is imminent.

Answer choice D is correct. In order to bring a claim in federal court, a plaintiff must have standing and the issue must be ripe for litigation. To have standing, a plaintiff must establish (1) a particularized injury, which must have actually occurred or be imminent; (2) that defendant caused this injury; and (3) that the relief requested is likely to redress the injury. First, the employer can show an imminent injury—loss of money from the threatened fines, which can be imposed immediately. Second, the government caused this injury by forcing the employer to comply with the law or pay a fine. Third, the requested injunction will redress that injury by removing the burden of compliance unless and until a court rules that the law is constitutional. Furthermore, the case is ripe because the threat of enforcement is imminent and the legal issues are sufficiently developed for a court to decide the case. Answer choice A is incorrect because neither of the categories of prudential standing applies here: The court is not being asked to adjudicate the claims of third parties or "generalized grievances" (i.e., injuries that are widely shared, as contrasted with the employer's individualized injury here). Answer choice B is incorrect because the two abstention doctrines—the Younger abstention doctrine, which applies only to pending state criminal cases or administrative proceedings, and the Pullman abstention doctrine, in which constitutional claims depend on resolving an unsettled issue of state law—are inapplicable. Answer choice C is incorrect because the doctrine of taxpayer standing, which allows a taxpayer standing to file a federal lawsuit challenging a specific legislative appropriation for violation of the Establishment Clause, is also inapplicable.

Question 3221 In order to save money, a state adopted a law restricting voting times and reducing the number of polling sites on Election Day. The law did not significantly impact the ability of voters to cast their ballots. The law applied to all state, local, and federal elections occurring on Election Day. Does Congress have the authority to override this law? Answers: No, because the voters' ability to cast their ballots was not significantly impacted. No, because the limitations are unrelated to the suppression of ideas. Yes, because the state cannot limit state or federal voting practices. Yes, because the state law regulates federal elections.

Answer choice D is correct. The Elections Clause explicitly empowers Congress to override state laws concerning federal elections. Here, the legislation attempts to regulate the federal election process by limiting voting times and polling sites as applied to federal elections, and Congress has the power to override such legislation. Answer choice A is incorrect because even if the voters are able to cast their votes, the limitations in the election process apply to federal elections. Congress has authority to override such a regulation. Answer choice B is incorrect because, even though the changes are unrelated to the suppression of ideas, Congress can nevertheless override state laws concerning federal elections. This power is not restricted to state election laws that suppress ideas. Answer choice C is incorrect because Congress does not have blanket authority to override a state election law that applies to elections for state offices.

Question 6276 A long-standing federal law provides for the payment of a $250,000 death benefit to the immediate family of a firefighter who dies as a direct result of fighting a fire. The law provides that this payment is in addition to any other benefit paid to the firefighter's family from another source, but does not otherwise address the relationship between this law and any conflicting state law. Acting pursuant to a recently enacted state law, a state worker's compensation commission reduced the amount of compensation paid to the widow of a firefighter by the amount of the federal death benefit. The widow challenged the constitutionality of the state law. How should the court rule on the challenge? Answers: Deny the challenge, because the commission's action did not affect the federal death benefit received by the firefighter's widow. Deny the challenge, because the federal law does not state that it expressly preempts conflicting state laws. Uphold the challenge, because the new state law was enacted after the long-standing federal law. Uphold the challenge, because the state law violates the Supremacy Clause of the United States Constitution.

Answer choice D is correct. While a federal law that expressly prohibits conflicting state laws is enforceable based upon the Supremacy Clause, the Supremacy Clause has also been interpreted as providing for implied preemption of a state law by a federal law when the two directly conflict. Here, the federal law specifies that the federal benefit is in addition to any benefits from another source, while the state law reduces the amount of the state worker's compensation payment by the amount of the federal benefit. For this reason, federal preemption is implied, and answer choice B is incorrect. Answer choice A is incorrect because, while the state law did not result in the direct reduction of the federal death benefit paid to the widow, the state law did conflict with the language in the federal statute stating that the federal death benefit was in addition to any other benefit from another source. Answer choice C is incorrect because federal preemption is not limited to conflicting state laws enacted after the passage of a federal law. Under the Supremacy Clause, any state law that directly conflicts with a federal law, even those state laws that predate the federal law, are preempted by the federal law.

Question 7417 Congress enacted legislation that made funding available to the states to provide services for children with special needs. The act conditions the funding on the states' meeting a variety of conditions designed to ensure that children with special needs received adequate services. In addition, the act grants parents of a child with special needs a right of action against a local school board for the failure to provide adequate services. The act permits the parents to recover the cost of tuition at a private school that meets the child's needs, together with "attorney fees and costs." The act is silent as to whether expert fees are considered costs. The parents of a child with special needs successfully pursued an action against the local school board under this statute. The court awarded the parents the appropriate damages, as well as attorney's fees and costs, including expert fees. Of the following, which is the most likely constitutional basis on which the local school board can challenge the court's award? Answers: The act itself is unconstitutional because the Tenth Amendment prohibits federal involvement in areas, such as elementary education, that have been traditionally reserved to the States. The awarding of expert fees is unconstitutional because conditions imposed on states by Congress in the exercise of its spending power must be set out unambiguously. The conditions imposed on the states to receive funding are unconstitutional, because Congress can only exercise its spending power to carry out another enumerated power. Providing the parents the right to bring this action is unconstitutional, because the Eleventh Amendment prohibits suits by private citizens against local governmental entities for money damages.

Answer choice B is correct. While Congress has the power under the spending power to spend for the general welfare and may impose conditions on the receipt of an appropriation by a state, such conditions must be set out unambiguously to be enforceable. Since expert fees were not unambiguously included in the amounts that may be recovered by the parents of a child with special needs under this statutory cause of action, these fees cannot be recovered. Answer choice A is incorrect. The Tenth Amendment provides that all powers not assigned by the Constitution to the federal government are reserved to the states, or to the people. However, the federal government has very broad authority under the spending power to spend for the general welfare, making state power rarely exclusive. Therefore, because states do not have exclusive power over education, this answer choice is incorrect. Answer choice C is incorrect. The spending power has been interpreted very broadly. Congress has the power to spend for the "general welfare"—i.e., any public purpose—not just to pursue its other enumerated powers. Therefore, this answer choice misstates the law. Answer choice D is incorrect. The Eleventh Amendment is a jurisdictional bar that prohibits the citizens of one state from suing another state in federal court. While the Supreme Court has expanded the amendment's reach to preclude citizens from suing their own state in federal court as well, this amendment does not apply to suits brought against local governments and their entities, such as school boards. Consequently, the provision in the act providing parents the right to sue their local school board does not violate the Eleventh Amendment.

Question 3259 U.S. armed forces seized an alien engaged in combat against them in a foreign country. The alien was taken to a territory outside the United States but over which the United States had sovereign control. Shortly thereafter, Congress passed a law denying federal courts jurisdiction over habeas corpus petitions filed by individuals who were designated as enemy combatants, but did not generally suspend the privilege of filing habeas corpus petitions. Subsequently, the alien, who was designated an enemy combatant by the President, filed a habeas corpus petition in federal court. May the court hear this petition? Answers: No, because Congress denied the federal courts jurisdiction over this type of petition. No, because Congress has plenary power over aliens. Yes, because Congress lacks the power to limit the jurisdiction of the federal judiciary. Yes, because the privilege of filing a habeas corpus petition extends to aliens in a territory over which the United States has sovereign control.

Answer choice D is correct. Under the Suspension Clause of Article I, Section 9, Clause 2, a detainee retains the privilege to file a habeas corpus petition unless this privilege is suspended. This clause applies to individuals detained in a territory over which the United States has sovereign control, even though such territory is outside the United States. Answer choice A is incorrect because, while Congress may limit the jurisdiction of the federal judiciary, Congress may not exercise this power in a manner that violates the Constitution. Accordingly, Congress may not limit the constitutional right to habeas corpus by removing jurisdiction from federal courts. Answer choice B is incorrect because, although Congress generally has plenary power over aliens, this power is subject to the privilege to file a habeas corpus petition. Answer choice C is incorrect because Congress has the power to limit the jurisdiction of the federal judiciary.

Question 1355 An unconstrued state law prohibited the distribution within the state of "seditious propaganda." The state prosecuted United States Post Office letter carriers under this law for delivering propaganda from a foreign country to state residents. Which of the following statements is an INACCURATE description of the state's law as applied to the letter carriers? Answers: It is an unconstitutional bill of attainder. It is void for vagueness. It may not be applied to the letter carriers, because they are employees of a federal instrumentality carrying out an authorized function. It unconstitutionally abridges rights protected by the First and Fourteenth Amendments.

Answer choice A is correct. A bill of attainder is a legislative act that inflicts punishment on named individuals or on easily identifiable members of groups. Because the statute proscribes a type of conduct when engaged in by anyone, it is not a bill of attainder. Answer choice B is incorrect because it accurately notes that a law is void for vagueness if it does not provide individuals with fair and reasonable notice of the boundaries between lawful and unlawful conduct. In this case, the statutory prohibition against "seditious propaganda" would be unconstitutionally vague under that standard, especially since the prohibition inhibits speech, a context in which the void-for-vagueness doctrine has special bite. Answer choice C is incorrect because it correctly applies the principle that federal employees are immune from state laws that interfere with their obligation to carry out an authorized function. Answer choice D is incorrect because it accurately concludes that the state law violates the constitutionally protected freedom of speech because it prohibits subversive speech without requiring the government to prove that the speech at issue was directed to inciting imminent unlawful action and was likely to do so.

Question 3201 A prominent broker was accused of insider trading. During his trial, he revealed the name of a colleague alleged to have provided him with the insider information. The colleague was subsequently proven to be innocent. In a ruling that was arguably incorrect per the state's court rules, the judge had allowed a live-feed of the court proceedings to occur on the Internet. As a result, the colleague's reputation was immediately ruined. His business was irrevocably damaged, and he was forced to file for bankruptcy. The colleague filed suit against the judge, alleging that had the judge not allowed court proceedings to be viewed on the Internet, his reputation and business would have remained intact. Will the colleague's lawsuit withstand a motion for summary judgment? Answers: No, because the judge is immune from liability. No, because it is not clear that the judge committed any error. Yes, because the damage to the colleague's reputation occurred outside of judicial proceedings. Yes, because the judge committed a grave procedural error.

Answer choice A is correct. A judge is absolutely immune from civil liability for judicial acts, including grave procedural errors. The judge is not immune, however, to lawsuits regarding non-judicial activities. The colleague filed a claim on the basis that the judge erroneously allowed the proceedings to be viewed by the public and should be liable for the damage that results. The judge is immune from such a suit. Answer choice B is incorrect because the judge would still be immune from civil liability even if he had committed an error, as the suit arises out of his judicial acts. Answer choice C is incorrect because while a judge is not immune from lawsuits regarding non-judicial activities, this harm resulted from the judge's decision regarding the conduct of the trial. The fact that the damage occurred outside the courtroom does not prevent the application of judicial immunity. Answer choice D is incorrect because even if the judge's action does constitute a grave procedural error, he would still be immune from civil liability.

Question 1382 A state required its political parties to allow every registered voter in the state to vote in party primaries. A newly-formed state party wanted to hold a primary to pick its presidential electors, but the national party with which the state party was affiliated required that electors be chosen only by party members. A neighboring state prohibited independents from voting in party primaries. A long-established third party hoped to finally secure the governor's mansion, which it felt hinged on the ability to attract a large percentage of the independent voters. Both the newly formed state party and the established third party challenged their respective states' primary regulations in federal court as unconstitutional. What will be the outcome of these lawsuits? Answers: Both the newly-formed state party and the established third party will prevail. The newly-formed party will prevail, but the established third party will lose. The established third party will prevail, but the newly-formed party will lose. Both the newly-formed party and the established third party will lose.

Answer choice A is correct. A state cannot require a local party to participate in an open primary (i.e., a primary in which any voter in the state may vote in a political party's primary) to choose presidential electors where the national party has required that electors must be chosen only by party members. In addition, a state may not prohibit a political party from allowing independents to vote in its primary. Answer choice B is incorrect because states cannot prohibit political parties from allowing independents to vote in primaries. Answer choice C is incorrect because open primaries cannot be required by the state. Answer choice D is incorrect because both statutes are unconstitutional. (Note: The length of time that the political party has been in existence is irrelevant for this purpose.)

Question 6610 The director of a community outreach program operated by a local government agency uncovered information that an employee of the program who was also a public official was grossly over reporting the number of hours she worked on her time sheets. The director fired the employee. Consequently, the employee became the subject of a criminal investigation that lead to her being charged with fraud and theft. In response to subpoenas, the director, who himself was an at-will employee, truthfully testified under oath both before a grand jury and at trial about the dismissed employee's falsified time sheets. Shortly thereafter, the director was fired in retaliation for his testimony. Can the director successfully seek reinstatement? Answers: Yes, because the director was fired in retaliation for speech made as a citizen about a matter of public concern. Yes, because the director testified pursuant to subpoenas. No, because the director was speaking as a public employee pursuant to his official duties about matters learned in the course of his employment. No, because the director was an at-will employee who may be dismissed without cause.

Answer choice A is correct. When a government employee contends that his rights under the Free Speech Clause of the First Amendment have been violated by his employer, the employee must show that he was speaking as a citizen on a matter of public concern. In determining whether a government employee is speaking pursuant to his official duties, the critical question is whether the speech at issue is itself ordinarily within the scope of an employee's duties, not whether it merely concerns those duties. When a public employee is speaking about a matter of public concern as a citizen rather than as employee, the First Amendment interest of the employee must be balanced against the interest of the state, as an employer, in effective and efficient management of its internal affairs. Here, the director was compelled to testify and did so truthfully. There is no evidence in the fact pattern to suggest that testifying in this manner was ordinarily within the scope of his duties. Rather, the testimony merely concerned information he learned in the course of his duties. Moreover, the fact pattern explicitly provides that his firing was done in retaliation for that testimony rather than on the basis of the government's needs as an employer. Consequently, the director can successfully seek reinstatement. Answer choice B is incorrect. Although the director was compelled to testify, he still could have been dismissed if his official duties typically involved testifying for the community outreach program. Here, the reason that the director may seek reinstatement is not because he was subpoenaed, but because he was fired in retaliation for protected speech as a citizen on a matter of public concern. Answer choice C is incorrect. In determining whether a government employee is speaking pursuant to her official duties, the critical question is whether the speech at issue is itself ordinarily within the scope of an employee's duties, not whether it merely concerns those duties. In this case, although the director was testifying about matters that he learned during the course of his employment, he was testifying in response to subpoenas as a citizen rather than as a public employee. Answer choice D is incorrect. Although an at-will public employee may not have a legitimate property interest in continued public employment and may generally be dismissed without cause, even an at-will employee may not be dismissed for reasons that in and of themselves violate the Constitution, such as in retaliation for protected speech.

Question 6592 A foreign corporation owned cargo containers that were used in international commerce. These containers were registered and subject to taxation on the basis of their full value in the corporation's home country. A state imposed an ad valorem tax on cargo containers owned by the foreign corporation that were found within the state on the annual tax day, as the state did for all such cargo containers found within the state on that day. The number of such containers owned by the foreign corporation in the state on tax day was a fair representation of the number of such containers that could be found within the state on any given day throughout the year. The foreign corporation has challenged in federal court the imposition of this ad valorem tax on its cargo containers as a violation of the Commerce Clause. How is the court likely to rule on this challenge? Answers: For the foreign corporation, because the tax subjected the corporation to international multiple taxation on the cargo containers. For the foreign corporation, because the tax was imposed by a state on foreign commerce. For the state, because the state applied the tax in a nondiscriminatory manner to all containers found within the state on tax day. For the state, because the tax was fairly apportioned based on the number of cargo containers owned by the corporation that could be found within the state on any given day.

Answer choice A is correct. A state tax imposed on interstate commerce must satisfy the Complete Auto test. Under this test, (i) the activity taxed must have a substantial nexus to the taxing state, (ii) the tax must be fairly apportioned, (iii) the tax may not discriminate against interstate commerce, and (iv) the tax must be fairly related to the services provided by the state. In addition to meeting the same requirements as a tax on interstate commerce, a state tax on foreign commerce must not (i) create a substantial risk of international multiple taxation or (ii) prevent the federal government from "speaking with one voice" regarding international trade or foreign affairs issues. Under these facts, because the cargo containers are subject to tax by the home country of the foreign corporation, the state tax subjects the containers to international multiple taxation, and thus the first of these two additional requirements has not been met. Consequently, the state tax on the cargo containers owned by the foreign corporation violates the Commerce Clause. Answer choice B is incorrect because a state is not prohibited by the Commerce Clause from taxing foreign commerce. However, in order to do so, the state tax must satisfy not only the Complete Auto test that applies to the taxation of interstate commerce but also two additional requirements imposed on state taxation of foreign commerce. Answer choice C is incorrect. Although the state ad valorem tax is applied in a nondiscriminatory manner and thereby satisfies one element of the Complete Auto test, this tax does not satisfy one of the additional requirements imposed on a state tax placed on foreign commerce. Answer choice D is incorrect because, although the state ad valorem tax was fairly apportioned and thereby satisfied the second element of the Complete Auto test, this tax does not satisfy one of the additional requirements imposed on a state tax placed on foreign commerce.

Question 7422 A state law created legislative districts for both houses of the state's legislature based on the total population of the state. The variation in people assigned to a district between the largest and smallest districts for the state senate was eight percent. However, due to the disproportionate distribution of eligible voters in the state, the variation in eligible voters assigned to the largest and smallest districts for the state senate was 40 percent. A voter in the district with the largest number of eligible voters filed suit contending that the legislative districts violate the Equal Protection Clause of the Fourteenth Amendment to the United State Constitution. Is the voter's challenge likely to succeed? Answers: No, because a state is permitted to draw state legislative districts based on total population. No, because a state senate district is not subject to the "one person, one vote" principle. Yes, because the districts do not comport with the "one person, one vote" principle. Yes, because even minor deviations in the size of state legislative districts are not permitted by the Equal Protection Clause.

Answer choice A is correct. Although the "one person, one vote" principle that is based on the Equal Protection Clause of the Fourteenth Amendment does apply to state legislative districts, a state may draw its legislative districts on the basis of total population rather than eligible or registered voters. In addition, a variation of less than ten percent in the size of a state legislative district is rebuttably presumed to be a minor deviation that does not constitute a prima facie case for discrimination. Answer choice B is incorrect because the "one person, one vote" principle applies to any state legislative district, even those districts that elect members to the state senate. Answer choice C is incorrect because, in the absence of evidence of discrimination, a variation of less than ten percent between the number of people based on total population in the smallest and largest districts does not establish a prima facie case for discrimination. Answer choice D is incorrect because, while near-mathematical precision is required between United States congressional districts, when the variation between state legislative districts is less than ten percent, it is presumed to be a minor variation that does not constitute a prima facie case for discrimination.

Question 6603 In order to protect valuable state resources from over-hunting, a state department enacted regulations establishing a lottery for permits to hunt elk in the state. The regulations put a cap on the percentage of permits granted to out-of-state residents. The regulations were challenged in court, and the federal appellate court found that the regulation violated the Dormant Commerce Clause and unconstitutionally discriminated against out-of-state commerce. In response to this decision, Congress passed legislation, which was signed by the President, that it was in the public interest for each state to continue to regulate the taking for any purpose of fish and wildlife within its borders, including by means of laws and regulations that differentiate between residents and nonresidents of the state. Subsequently, another state adopted a regulation that prohibited out-of-state commercial hunters from hunting elk within the state. A group of out-of-state residents who lead elk hunting expeditions sued the state official responsible for enforcing the regulation, seeking a declaratory judgment that the regulation violated the Dormant Commerce Clause. If the court determines that hunting is an economic activity that substantially affects interstate commerce, should the court grant the requested declaratory judgment? Answers: No, because Congress granted the states the right to discriminate against out-of-state residents. No, because hunting is a traditional area of state regulation. Yes, because the federal law improperly invades the province of the court. Yes, because the regulation discriminates against out-of-state residents.

Answer choice A is correct. Because Congress has exclusive authority over interstate commerce, it may explicitly permit states to act in ways that would otherwise violate the Dormant Commerce Clause. It must be unmistakably clear that Congress intended to permit the otherwise impermissible state regulation. Here, Congress has specifically provided that states may, through regulations as well as statutes, discriminate against out-of-state commercial hunters with regard to hunting and fishing within the state. Therefore, the regulation is permitted. Answer choice B is incorrect. Although hunting is an area that traditionally has been subject to regulation by the states, Congress may regulate economic activity that substantially affects interstate commerce. Consequently, the Tenth Amendment, which reserves to the states only those powers not delegated to the federal government or denied to the states, does not serve to permit this regulatory restriction on out-of-state hunters. Answer choice C is incorrect. Although the judiciary has the final say in interpreting the Constitution, here, Congress is engaged in its law-making function pursuant to the Constitution to regulate interstate commerce. Answer choice D is incorrect. Although the regulation clearly discriminates against out-of-state residents, Congress has expressly permitted this discrimination.

Question 6293 In order to discourage the transfer of electrical products that could threaten the security of the United States as well as to raise revenue, Congress enacted a statute that imposed a tax on the export of specific electrical products to countries that were determined to be hostile to the United States. Is this statute constitutional? Answers: No, because of the Export Taxation Clause of Article I, Section 9 of the United States Constitution. No, because Congress has no general police power to legislate for the health, safety, welfare, or morals of citizens. Yes, because of the taxing power granted to Congress in Article I, Section 8 of the United States Constitution. Yes, because of the Commerce Clause of Article I, Section 8 of the United States Constitution.

Answer choice A is correct. Congress may not tax goods exported to foreign countries. Under the Export Taxation Clause, a tax or duty that falls on goods during the course of exportation or on services or activities closely related to the export process is prohibited. Answer choice B is incorrect. While Congress, unlike a state legislature, does not enjoy a general police power, Congress can generally exercise its power to tax for any public purpose, such as national security. However, the Export Taxation Clause specifically prohibits Congress from taxing exports. Answer choice C is incorrect. Although Article I, Section 8 of the Constitution gives Congress the plenary (i.e., exclusive) power to raise revenue through the imposition of taxes, and the General Welfare Clause of this section has been interpreted as permitting Congress to exercise its power to tax for any public purpose, the Export Taxation Clause specifically prohibits Congress from taxing exports. Answer choice D is incorrect. Although the Commerce Clause grants Congress power over commerce, including international commerce, the Export Taxation Clause specifically prohibits Congress from taxing exports.

Question 4178 An employee of a large non-denominational church served five years as the minister for the church's youth program. During the past year, the minister was diagnosed with severe chronic migraine headaches, which caused him to miss work one or two days per week. Subsequently, the senior pastor of the church and the board of deacons, who had final say on the hiring and firing of all ministerial staff, unanimously voted to terminate the minister's employment and provide him with a modest severance package. He filed suit against the church under the Americans with Disabilities Act, which, in part, protects both government and private employees against discrimination based on certain medical and physical disabilities, including chronic migraine headaches. The minister seeks reinstatement and back pay. The church filed a motion for summary judgment. Should the court grant the church's summary judgment motion? Answers: Yes, because the Free Exercise clause of the First Amendment supports the dismissal. Yes, because it violates the church's First Amendment association rights. No, because the Americans with Disabilities Act is a neutral law of general applicability. No, because the minister's due process and equal protection rights were violated.

Answer choice A is correct. In Hosanna - Tabor Evangelical Church and School v. EEOC, 132 S.Ct.694 (2012), the Supreme Court unanimously recognized the existence of a "ministerial exception" grounded in the First Amendment Religious Clauses that precluded the application of employment discrimination laws concerning the employment relationships between a religious organization and its ministers. Requiring a church to accept or retain an unwanted minister interferes with the internal operations of a religious organization in violation of the Free Exercise and Establishment Clauses of the First Amendment. Answer choice B is incorrect. The Supreme Court has rejected the argument that the ministerial exception violated the free association rights of a church under the First Amendment. The text of the First Amendment gives "special solicitude" to the rights of religious organizations. Answer C is incorrect, because even though the Americans with Disabilities Act is a neutral law of general applicability, the ministerial exception precludes the application of this law to a religious organization's minister. Answer choice D is incorrect because an employment claim by a minister is not saved by either the Due Process or Equal Protection clauses. The "ministerial exception" trumps other constitutional arguments.

Question 5943 A federal statute mandates that any entity employing 50 or more individuals must derive at least five percent of its energy needs from renewable resources. The Department of Energy (DOE) has been tasked with the enforcement of this statute. A county government that employs more than 50 individuals has sued the DOE official to whom enforcement of this statute has been properly delegated in the federal district court. The suit, which seeks injunctive relief from the enforcement of this statute, contends that the statute is unconstitutional as applied to the county government. Should the court rule that the statute is unconstitutional as applied to the county government? Answers: No, because the statute does not commandeer county officials to regulate the conduct of others. No, because the federal government is immune from suit by the county government. Yes, because the statute violates the Tenth Amendment. Yes, because the statute violates the Eleventh Amendment.

Answer choice A is correct. In general, the federal government has almost unlimited power to regulate the states as long as Congress is exercising one of its enumerated powers, such as the power to regulate commerce. The primary limitation on Congressional action with regard to the states is that Congress may not commandeer state legislatures to enforce a federal regulatory program, either directly or by conscripting a state official. This does not mean that a county government cannot be subject to the federal statute, only that Congress cannot force the county government to use its power and resources to enforce the federal program in the county. Here, because the statute in question does not seek to commandeer either the state legislature or a state official, the statute should survive the county's constitutional challenge. Answer choice B is incorrect. Although the doctrine of sovereign immunity generally prevents the federal government from being sued without its consent, a federal official, such as the DOE official in this case, may be sued for injunctive relief. Answer choice C is incorrect because the Tenth Amendment has not been interpreted as imposing a limitation on the exercise of Congressional power that affects state or local governments or officials. Answer choice D is incorrect because the Eleventh Amendment does not impose a restriction on the federal government, but instead protects the states from certain lawsuits brought against the state by out-of-state citizens in federal court.

Question 1359 A state law that restricted abortion was challenged in state court as a violation of the Due Process Clause of the Fourteenth Amendment to the U.S. Constitution and as a violation of a similar due process provision of the state constitution. The case made its way to the state's highest court, which ruled that the law violated the due process provisions of both the U.S. and the state constitutions. If petitioned to do so, may the U.S. Supreme Court exercise jurisdiction to review the state court decision? Answers: No, because the state court's decision in this case rests on adequate and independent state law grounds. No, because the U.S. Supreme Court has appellate jurisdiction only over state court decisions that determine the constitutionality of federal laws. Yes, because the U.S. Supreme Court has appellate jurisdiction over any ruling of a state's highest court based on an interpretation of federal law. Yes, because the U.S. Supreme Court has appellate jurisdiction over decisions that find state laws in violation of the federal Constitution.

Answer choice A is correct. The U.S. Supreme Court does not have appellate jurisdiction over a decision by the highest court of a state when that decision is supported by state law grounds that are (1) independent of federal law and (2) adequate to sustain the result in the case. The result in this case is fully supported by the state court's ruling that the law violated the state constitution, and this ruling is independent of any consideration of the federal constitutional claim. Accordingly, even if the U.S. Supreme Court were to reverse the state court's ruling on the federal constitutional issue, the result in the case would not change, and thus the decision is not reviewable. Answer choice B is incorrect because while this response correctly states that the U.S. Supreme Court may not review the decision, it inaccurately states the scope of the Court's appellate jurisdiction. Answer choice C is incorrect because the U.S. Supreme Court may not review this decision. While the state court opinion did cite a violation of federal law, it also stated that the law violated the due process clause of the state constitution. The opinion therefore reflects sufficiently adequate and independent state grounds preventing the U.S. Supreme Court from reviewing the case. Answer choice D is incorrect because while the statement in this response is generally true, the U.S. Supreme Court does not have appellate jurisdiction over a decision of the highest court of a state finding a state law in violation of the federal Constitution when that decision is supported by state law grounds that are (1) independent of federal law and (2) adequate to sustain the result in the case. EDITOR'S NOTE: This case can be distinguished from Michigan v. Long. In that case, the state court's decision rested almost exclusively on the interpretation of federal law; here, the state court has ruled that the state constitution had been violated.

Question 432 A city police officer sought and received an anticipatory warrant to search the defendant's premises for evidence of a crime. The defendant, seeking to suppress the evidence seized by the officer during the search, challenged the legality of the warrant. The highest court in the state, while noting that the federal Constitution permits anticipatory search warrants, clearly ruled that the state constitution, which contains language that is similar to the Warrant Clause of the Fourth Amendment, does not. The state has appealed this ruling to the United States Supreme Court. Which of the following actions must the Supreme Court take? Answers: Refuse to hear the appeal. Reverse the state court decision, because the federal Constitution permits anticipatory warrants. Uphold the state court decision, because the federal Constitution does not require anticipatory warrants. Uphold the state court decision, because the issue involves a criminal matter, an area traditionally left to the states under the Tenth Amendment.

Answer choice A is correct. The federal Constitution establishes a floor, but not a ceiling, on individual rights. A state is free to interpret its own constitution as granting greater protection to persons than the United States Supreme Court has interpreted the United States Constitution as granting. For this reason, answer choices B and C are incorrect. A final judgment issued by a state court that clearly rests on adequate and independent state grounds is not reviewable by the United States Supreme Court. To do otherwise would require the Supreme Court to issue an advisory opinion. Answer choice D is incorrect because the Tenth Amendment does not prohibit the United States Supreme Court from deciding criminal issues where federal rights are implicated.

Question 7055 State A recently enacted a statute requiring water-bottling facilities in the state to use only plastic water bottles produced in State A. The stated purpose of the statute is to encourage the use of a specific type of plastic that is easier to recycle. State A has multiple manufacturers that produce this specific type of plastic water bottle. Before the statute was enacted, State B was a major supplier of plastic water bottles used in State A water-bottling facilities. State B also produces many types of plastic water bottles, including the specific type that is easier to recycle. A plastic bottle manufacturer in State B who produces this specific type of plastic water bottle has challenged the constitutionality of the statute. Is the statute constitutional? Answers: No, because a state statute that favors local businesses over out-of-state competitors is always invalid. No, because the interest served by the statute could be accomplished by nondiscriminatory means. Yes, because an important local interest regarding a type of plastic that is easier to recycle is being served. Yes, because the Dormant Commerce Clause does not prevent a state from regulating intrastate businesses.

Answer choice B is correct. A state or local regulation discriminates against out-of-state commerce if it protects local economic interests at the expense of out-of-state competitors. Here, the statute is protecting the local manufacturing of plastic bottles at the expense out-of-state competitors. Because other nondiscriminatory means are available to achieve that purpose (e.g., a statute requiring the use of the specific plastic rather than just requiring bottles made in State A), this discriminatory statute is invalid under the Dormant Commerce Clause. Answer choice A is incorrect because it is overly broad. A discriminatory regulation may be upheld if the state or local government can establish that an important local interest is being served, and no other nondiscriminatory means are available to achieve that purpose. Answer choice C is incorrect because an important interest is insufficient to allow a statute discriminating against out-of-state commerce to succeed. The statute must also have no nondiscriminatory alternative to achieve the interest. Answer choice D is incorrect. State regulations that regulate intrastate businesses can still burden or discriminate against interstate commerce. Here, even though the statute only applies to bottling facilities in State A, its requirement that these facilities only purchase bottles manufactured in State A discriminates against out-of-state commerce and thus implicates the Dormant Commerce Clause.

Question 5887 A political activist is opposed to same-sex marriages. Recently, the state where the activist lives passed a statute criminalizing "any action that threatens violence and is motivated by the intent to cause fear of violence." A same-sex couple planned on holding a legal marriage ceremony on the beach with close family and friends. The activist learned of the marriage ceremony and decided to attend the event in order to stop the marriage. At the marriage ceremony before the couple uttered their vows, the activist screamed to the couple, "If you get married, I will drown both of you in the ocean." The couple feared for their safety, believing that the activist would drown them if they continued the ceremony. However, the activist was arrested before he could take any action, and he now claims that he did not intend to drown the couple; his only intention was to disrupt the marriage ceremony. Can the activist be convicted under the statute? Answers: Yes, because the couple feared for their safety. Yes, because the activist threatened to drown the couple in the ocean. No, because the man did not intend to drown the couple. No, because the activist's words were simply annoying and offensive.

Answer choice B is correct. Actual threats of violence are outside the protection of the First Amendment. Here, the activist threatened to drown the couple if they persisted in their marriage vows. This threat was communicated with the intent to intimidate the couple, so it is not protected by the First Amendment. Answer choice A is incorrect. Although the couple feared for their safety, such fear is not required in order to convict the activist under the state statute. The criminal act is in making the threat with the intent to cause fear of violence. Answer choice C is incorrect. Even if he did not intend to actually drown the couple, the activist intended to cause fear of violence by threatening to drown the couple in the ocean. This threat is not protected by the First Amendment and the activist can be convicted under the state statute. Answer choice D is incorrect. Here, the activist's words were not simply annoying and offensive. They constituted a threat of violence and were intended to cause fear of violence.

Question 7045 In general, the beef industry has been gravitating towards grass-fed cows instead of corn-fed cows, for reasons of health and taste. Grass fields that grow hay to feed grass-fed cows are currently located in only 10 states. In order to capitalize on this growing market as a source of revenue, Congress recently enacted a statute placing a high per-unit sales tax on the type of grass used for hay. The tax will discourage cow farmers from feeding their cows grass instead of corn because of the significant increase in the cost of grass over corn. Additionally, the increased cost will eventually be passed on to consumers, who analysts predict will be less inclined to choose expensive grass-fed beef over cheaper corn-fed beef. If the farmers who harvest and sell this grass file a lawsuit in federal court challenging the constitutionality of this sales tax, are they likely to succeed? Answers: No, because Congress can enact any legislation that is necessary and proper to execute its authority to produce revenue. No, because Congress has plenary power to impose and collect taxes if they are reasonably related to revenue production. Yes, because application of the tax will not be apportioned properly between the states that have the necessary grass fields and those that do not. Yes, because the tax violates the uniformity requirement of Article I, Section 8 of the Constitution.

Answer choice B is correct. Article I, Section 8 of the Constitution gives Congress the power to levy and collect taxes. A tax by Congress will generally be upheld if it has a reasonable relationship to revenue production, or if Congress has the power to regulate the activity being taxed. The sales tax on this variety of grass is reasonably related to revenue production, and there do not appear to be any provisions extraneous to tax purposes. Thus, the tax is likely constitutional. Answer choice A is incorrect because the Necessary and Proper Clause is not an independent source of power; it merely gives Congress the power to execute specifically granted powers. Thus, it cannot be the only enabling clause cited to justify the imposition of a tax. Answer choice C is incorrect because only direct taxes, such as ad valorem property taxes, are subject to the apportionment requirement. Answer choice D is incorrect because the requirement that taxes be uniform throughout the United States means only that the product must be identically taxed in every state in which it is found. Here, that requirement will be met.

Question 7059 City officials recently received a study showing a sharp increase in underage drinking. The study suggested that most purchases of alcohol by minors occurred at stores within 800 feet of a public high school, and were of a certain type of alcoholic beverage marketed to young adults. In response to the study, the city enacted a zoning ordinance that forbade the sale of that alcoholic beverage within 800 feet of a public high school. The owner of a liquor store 700 feet away from a public high school challenged the constitutionality of the zoning ordinance on the grounds that the zoning ordinance was too restrictive a solution, and that stricter enforcement of photo-ID laws would be a less restrictive and more effective solution to the problem. The owner requested an injunction preventing enforcement of this regulation against his store. If the owner fails in his challenge of the zoning ordinance, what is the most likely reason? Answers: There is no constitutional limitation on enacting zoning ordinances that do not impact a fundamental right. Reasonable ordinances may restrict nonfundamental rights to further a legitimate governmental interest. There was no less restrictive regulation that would have addressed the same governmental interest more effectively. The government may incidentally block adult access to otherwise legal substances and materials to keep them out of the hands of minors.

Answer choice B is correct. Government infringement upon nonfundamental rights—those related to social or economic interests such as business, taxation, lifestyle, or zoning—requires only a rational relationship between the law and a legitimate governmental interest. Here, the narrowly drawn zoning ordinance was constitutional because the purpose of the regulation was to reduce the sharp increase of underage high-school students from purchasing specific alcoholic beverages within 800 feet of public high schools. Limiting the sales of such beverages within this distance is rationally related to this interest. Therefore, this ordinance is most likely constitutional for this reason. Answer choice A is incorrect because the government does not have the unfettered right to enact zoning ordinances. Even ordinances that do not affect a fundamental right must still be rationally related to a legitimate government interest. Answer choice C is incorrect because this land-use restriction is tailored in response to the fact that a substantial amount of the alcohol purchased by minors was of a specific type and occurred within 800 feet of a public high school. The mere existence of an alternative does not conclusively establish that the regulation enacted by the city is not sufficiently related to a legitimate governmental interest. Therefore, this suggested alternative is insufficient to make the zoning regulation unconstitutional. Answer choice D is incorrect. It misstates the law regarding obscene materials, which states that the government cannot block an adult's access to indecent materials to keep them from reaching minors.

Question 7419 A city transferred ownership of public streets within a housing development to the city's housing authority. The city council then adopted an ordinance declaring these streets private property under the control of the housing authority. Pursuant to that authority, the housing authority posted "no trespassing" signs and authorized the city's police to serve a barment notice on any person who could not demonstrate a "legitimate business or social purpose" for being on the premises. The barment notice forbade the person from returning to the property. The housing authority also authorized the city's police to arrest any person who disobeyed a previous barment notice for trespassing. A person walking through the housing authority property received a barment notice and was subsequently arrested by city police for trespassing after being found on the property a second time. The person has challenged his conviction on the grounds that the housing authority's policy violates the First Amendment on its face as an overbroad restriction on speech. Of the following, which is the least likely basis upon which the court would deny this challenge? Answers: The person has not met his burden of establishing that the housing authority's policy is overbroad. The person lacks standing to challenge the housing authority's policy on the grounds of overbreadth. The policy does not affect a substantial amount of constitutionally protected speech. The policy does not directly punish speech or conduct related to speech.

Answer choice B is correct. In order to prevent a "chilling effect" on protected speech, an overbroad governmental law or policy may be challenged as facially invalid even by those who are validly regulated on behalf of those who are not. Here, the streets could be considered a traditionally public forum, and the housing authority's policy is broad enough that it could limit protected speech by others on these streets. Consequently, even though the person cannot bring this action on his own behalf because he was not engaged in protected speech, he may assert an overbreadth complaint on behalf of those who would be. Answer choice A is incorrect because the burden is on the person who challenges a governmental policy or law as being unconstitutionally broad; therefore, the failure to meet this burden is a ground on which this challenge can be denied. Answer choice C is incorrect because it does not appear that the housing authority's policy affects a substantial amount of constitutionally protected speech, so the court can reject the challenge on this basis. Answer choice D is incorrect because, since the policy is not aimed at punishing speech or conduct related to speech, the court can reject this challenge on this basis.

Question 6275 Congress enacted and the President signed the Toxic Waste Policy Act (Act). The Act delegated the primary responsibility for developing and administering a toxic waste disposal program to the Department of Energy (DOE) based on intelligible standards set forth in the Act. The Act also authorized the creation of a toxic waste fund to impose the cost of disposing of toxic waste on the creators of such waste through a statutorily-fixed fee. A provision of the Act requires the Secretary of the DOE to annually evaluate the fees collected to ensure that those fees offset the costs of the waste disposal program. If not, the Secretary is required to adjust the amount of the fee accordingly. The Act provides that the Secretary's adjusted fee becomes effective 90 days after publication unless both Houses of Congress adopt a resolution disapproving of the adjustment. This year the DOE Secretary, pursuant to the provision, proposed a fee increase. Both Houses of Congress disapproved the increase by resolution. Is a challenge to constitutionality of this provision likely to be successful? Answers: Yes, because the Act violates the "nondelegation doctrine." Yes, because the provision provides for a "legislative veto" of an executive action. No, because the principle of bicameralism has been satisfied. No, because the President approved of the provision by signing the Act.

Answer choice B is correct. It is unconstitutional for Congress to legislatively veto an executive action—that is, to retain direct control over the actions of an executive agency, rather than going through the proper channels of passing a bill. Answer choice A is incorrect. Although the "nondelegation doctrine" provides that Congress may not delegate legislative power to any other branch of government because Congress is vested by Article I with "all legislative powers," the delegation of some of Congress's authority to the executive branch has consistently been held constitutional, so long as Congress specifies an "intelligible principle" to guide the delegation. Since the Act contains intelligible standards by which the DOE is to develop and administer the toxic waste disposal program, it does not violate this doctrine. Answer choice C is incorrect because, although the passage of the resolution by both Houses of Congress does satisfy the principal of bicameralism, the provision nonetheless runs afoul of the Article I requirement that bills passed by both chambers of Congress must be presented to the President for signature or veto. Answer choice D is incorrect because the federal judiciary retains the power to declare a federal statute unconstitutional, even though the statute has received the approval of the other two branches of government (the executive as well as the legislative branches), and even though the President has agreed to the limitation on executive authority.

Question 4366 A city enacted ordinances that regulated new construction within a community on the edge of the city but still within the city's boundaries. The ordinances required that any new construction within a ten-block radius of the main street in the community must include retail establishments on the first level. Based on multiple studies, the city knew that current city-dwellers lived in the city for the outstanding amenities, including entertainment and dining establishments. The same studies also revealed that current city-dwellers were dissatisfied with severe traffic problems and overcrowding within the city. The ordinances were enacted by the city to encourage current city-dwellers to relocate to the community by creating a commercial district in the community without the challenges of overcrowding and traffic. After enacting a similar statute on a much smaller scale, the city had seen a decrease in city traffic and overcrowding and knew it could replicate these results on a larger scale. The head of a construction company files suit, alleging that the ordinance prevents him from being able to construct a planned apartment complex in accordance with its current design. The city defends the application of the ordinances on the basis that busier areas, including commercial districts, are generally much safer. Is the city's defense likely to be successful? Answers: Yes, because the city does not need a legitimate purpose to enact zoning ordinances. Yes, because the ordinance is related to a legitimate state interest. No, because the initial reason for enacting the statute was to reduce traffic and overcrowding. No, because the city violated the construction company's due process rights.

Answer choice B is correct. Most legislation related to zoning is reviewed under the rational basis standard, and a law meets that standard if it is rationally related to a legitimate state interest. Laws are presumed valid under this standard, so the burden is on the challenger to overcome this presumption by establishing that the law is arbitrary or irrational. In court, the government's stated interest in enacting the law need not be one that it offered when the law was passed, and here, the city's stated interest constitutes a legitimate interest. The ordinances, while not necessarily the most direct way to accomplish that interest, are rationally related to it. Answer choice A is incorrect because the city does need to meet the rational basis standard when enacting laws related to zoning, though the burden falls on the challenger to demonstrate that the law is not rationally related to the interest. Answer choice C is incorrect because the government interest need not be stated when the law was passed, as long as the stated interest is legitimate. Answer choice D is incorrect because the zoning ordinance does not violate the company's due process rights because it passes the rational basis test.

Question 3198 A federal law provides that all employees of the federal library system whose job descriptions include cataloging or disseminating materials will be required to take a reading comprehension test. The law provides that employees who do not meet a minimum standard of reading comprehension will either be fired or reassigned to a different position. The law does not provide for a hearing process for these employees, who are not at-will employees. An employee whose job description included cataloging materials failed to meet the minimum standard and was reassigned to a different position at a lower grade. The government denied his request for a hearing. He sued to enjoin the government from reassigning him, arguing that his constitutional rights were violated. Which of the following constitutional provisions provides the best support for the employee's suit? Answers: The Privileges and Immunities Clause of Article IV. The Due Process Clause of the Fifth Amendment. The Takings Clause of the Fifth Amendment. The Contract Clause of Article I, Section 10.

Answer choice B is correct. The Due Process Clause of the Fifth Amendment provides that the federal government shall not deprive any person of "life, liberty, or property, without due process of law." An employee who is not at-will has a legitimate property interest in continued public employment, and is entitled to a fair procedure to protect the employee's interests if the government seeks to discharge the employee from his position. An employee is entitled to a notice of termination and a pre-termination opportunity to respond, which the employee in this case did not receive. Answer choice A is incorrect because the Privileges and Immunities Clause prohibits one state from discriminating against the citizens of another state, and thus is not applicable in this case. Answer choice C is incorrect because the Takings Clause generally applies to the government's taking of an individual's real property, rather than an employment interest. Answer choice D is incorrect because Article I, Section 10 prohibits states from passing laws that impair the obligations of contracts; it does not apply to the federal government.

Question 3211 A small, struggling, formerly industrial city in one state was only 30 miles from the border of the neighboring state, and only 40 miles from a booming city in the neighboring state. In an effort to entice more citizens of the neighboring state to come to the small city to shop, dine, and otherwise spend money, the small city passed an ordinance relieving out-of-state citizens from paying the city's sales tax. A group of small-city citizens properly brought a suit against the city, challenging the ordinance. Of the following constitutional provisions, which would be the basis on which the citizens could most effectively challenge the ordinance? Answers: The Due Process Clause of the Fifth Amendment. The Equal Protection Clause of the Fourteenth Amendment. The Privileges and Immunities Clause of Article IV, Section 2. The Privileges or Immunities Clause of the Fourteenth Amendment.

Answer choice B is correct. The Equal Protection Clause of the Fourteenth Amendment provides that "no state shall...deny to any person within its jurisdiction the equal protection of the laws." The Equal Protection Clause is generally the constitutional safeguard that a citizen or group of citizens will use to challenge a law that is based on some kind of classification. Here, the city is classifying persons as either state citizens or out-of-state citizens, and providing the latter group with a benefit by relieving them from paying the city's sales tax. Therefore, the citizens could challenge this law on equal protection grounds. Answer choice A is incorrect because the Due Process Clause of the Fifth Amendment applies to the federal government, not to state action. Answer choice C is incorrect. The Privileges and Immunities Clause of Article IV prevents a state from treating citizens of another state in a discriminatory manner; it does not prevent a state from discriminating against its own citizens. In this case, the state is treating out-of-state citizens in a more advantageous manner than it is treating its own citizens; it is not discriminating against them. In-state residents have no claims against their own state under the Privileges and Immunities Clause. Answer choice D is incorrect because the Privileges or Immunities Clause of the Fourteenth Amendment has very limited application. This provision prevents states from infringing on the rights of national citizenship, and is usually only successfully invoked in cases involving the right to travel between the states.

Question 6608 A federal statute that governs employer-provided health benefits specifies that health maintenance organizations (HMOs) may limit their provider networks. The statute also provides that state law that regulates insurance is not preempted by the statute. Federal courts interpreting the statute have held that in order to qualify as a law that regulates insurance, a state law must be specifically directed towards entities that engage in insurance. A state law required health insurers to acknowledge the services of any health care provider willing to abide by the insurer's plan. The HMO filed an action in federal court contending that, pursuant to federal law, it could limit its provider networks. The federal court, noting that a state court had interpreted the state law as applying to HMOs, rejected the HMOs argument that the state law did not regulate insurance. Which of the following states the reason why the state law is constitutional under the Supremacy Clause of Article VI of the U.S. Constitution? Answers: Under the Supremacy Clause, a state law is not preempted by a federal law unless the federal law does so expressly. Under the Supremacy Clause, the federal statute governs the enforceability of the state law. The Supremacy Clause requires the federal court to accept a state court's interpretation of state law. The Supremacy Clause does not apply to this state law because the state law regulates insurance.

Answer choice B is correct. The Supremacy Clause of Article VI provides that the "Constitution, and the laws of the United States" are the "supreme law of the land." Any state constitutional provision or law that directly or indirectly conflicts with a federal law, including federal regulations, is void under this clause. However, in this instance, the federal statute specifically disavowed federal preemption of a contrary state law if the state law regulated insurance. Because the federal court rejected the HMO's argument that the state law did not regulate insurance, the state law was constitutional by virtue of the federal statute's anti-preemption provision. Answer choice A is incorrect because a state law may be preempted by implied as well as express federal preemption, such as when federal law occupies an entire area of the law. Answer choice C is incorrect because the Supremacy Clause does not govern the extent to which a federal court must respect a state court's interpretation of its own laws. Here, the federal court, under principles of federalism, must defer to the state court's interpretation of state law as applying to HMOs. However, the determination of whether the state law in question regulates insurance involves interpretation of a federal, not state, statute. Answer choice D is incorrect because the Supremacy Clause applies to all state laws, even ones that regulate insurance. However, in this case, there is a federal statute that contains an express anti-preemption provision for state laws that regulate insurance.

Question 4361 Several cities within a state passed ordinances that prohibited discrimination against gay and lesbian individuals in employment and housing. In response, the state legislature passed a statute that prohibited any city in the state from passing any ordinance that recognized gay or lesbian individuals as a protected class. A resident of the state sued the state, arguing that the statute violated the Equal Protection Clause. Is the law likely to be upheld? Answers: No, because the Supreme Court has recognized sexual orientation as a suspect classification. No, because the law would not satisfy the rational basis test. Yes, because sexual orientation is not a suspect classification subject to the Equal Protection Clause. Yes, because the law would satisfy the intermediate scrutiny test.

Answer choice B is correct. The Supreme Court has struck down bans on same-sex marriage as violations of a fundamental right on both Due Process and Equal Protection grounds, but it has not resolved the issue of whether discrimination based on sexual orientation is subject to heightened scrutiny. Regardless of whether heightened scrutiny applies here, the government cannot impose a burden upon or deny a benefit to a group of persons solely based on animosity toward the class that it affects. In this case, a court would likely strike down the law because it does not satisfy the rational basis test. Answer choice A is incorrect because sexual orientation is not a suspect classification. Answer choice C is incorrect because, although sexual orientation is not a suspect classification, laws based on such a classification are subject to Equal Protection analysis. Answer choice D is incorrect because the intermediate scrutiny test, which is used for classifications based on gender or status as a nonmarital child, has not been found to apply to sexual orientation.

Question 6299 A professional basketball player who was a citizen of one state sued an artist who was a citizen of another state. The artist had created multiple oil paintings of the player's image from which limited edition prints were created and sold without the player's consent or permission. The player sought damages in excess of $80,000 for violation of his state statutory right to publicity. While the state trial court ruled in the player's favor, the state appellate court overturned this decision on the erroneous grounds that a free speech privilege arising under the First Amendment protected the artist from this action. The appellate court decision denied the player recovery on his state-based cause of action. The state's highest court declined to hear the appeal. The player filed a petition for writ of certiorari with the U.S. Supreme Court. Can the Supreme Court grant this petition? Answers: Yes, because diversity of citizenship exists between the player and the artist and the amount in controversy exceeds $75,000. Yes, because the state court decision turned on whether a federal constitutional privilege existed for the artist. No, because the state's highest court did not render a decision in this case since it declined to hear the appeal. No, because the player sought to recover damages from the artist under a state statutory cause of action.

Answer choice B is correct. The Supreme Court's only power over a state court judgment is to correct the judgment when the state court incorrectly applies federal rights. Here, the Supreme Court can grant this petition because the state court judgment turns on the erroneous grounds that there is a federal constitutional privilege that protected the artist from a right to publicity cause of action. Answer choice A is incorrect because, while satisfaction of the diversity of citizenship requirements is one path by which a federal district court can exercise jurisdiction, it is not a requirement in order for the Supreme Court to exercise appellate jurisdiction over a state court decision. Answer choice C is incorrect because, while the Supreme Court generally reviews only decisions made by the highest court of a state, there is no constitutional requirement that the Supreme Court do so. Moreover, in this instance, the decision by the state's appellate court represented the final word on the matter by the state judiciary, since the state's highest court has refused to review that decision. Answer choice D is incorrect. While the state court decision in the case brought in state court under a state statutory cause of action generally would rest on adequate and independent state grounds that would foreclose review of the state court decision by the Supreme Court, in this case, the state court decision turns on whether a privilege based on the federal Constitution exists.

Question 5947 Concerned with the proliferation of signs about upcoming events and the failure to remove those signs after the event, a city enacted an ordinance that limited the number of such signs that could be displayed on public property and set a time period before and after an event during which signs about that event could be displayed. A social organization wants to display signs about its monthly dinner, which is held to attract new members, in greater number and for a longer period than permitted by the ordinance. The organization has filed a lawsuit, challenging the constitutionality of the ordinance. Of the following, by which standard will this ordinance be judged? Answers: It must be narrowly tailored to further a significant government interest, and leave open alternative channels of communication. It must be the least restrictive means for accomplishing a compelling governmental interest. It must be rationally related to a legitimate government interest. It must not have a negative impact on the organization's freedom of assembly.

Answer choice B is correct. This regulation is a restriction on speech that is content-based on its face. The regulation applies to one specific topic (upcoming events) and subjects that speech to different--and more onerous--requirements than any other signs. Therefore, the city's ordinance will be judged by a strict scrutiny standard. Answer choice A is incorrect because it states the standard for time, place, and manner restrictions. However, this is a content-based restriction, so strict scrutiny applies. Answer choice C is incorrect because it states the rational basis test, which does not apply to content-based restrictions on speech. Answer choice D is incorrect. All constitutional protections, including the First Amendment protection of the freedom of assembly, are not absolute. The mere fact that governmental conduct has a negative impact on this right is not sufficient to preclude the constitutionality of an ordinance.

Question 3214 A federal statute provides that a U.S. territory can freely trade certain agricultural products, including bananas, with a foreign island nation. Shortly after the enactment of this statute, a U.S. state enacted a law allowing for a free transfer of goods between the state, the territory, and the island nation. Several years later, the President of the United States entered into a self-executing treaty with several trading partners that in pertinent part required a mark of origin on all agricultural products from all foreign countries. The Senate promptly ratified this treaty. Last year, the President of the United States entered into an executive agreement with the leader of a second foreign country, prohibiting the importation into the United States of bananas from any other country. Citizens of the U.S. territory want to continue to freely buy bananas from the island nation. Which of the following is likely to govern whether citizens of the U.S. territory can buy bananas from the island nation? Answers: The executive agreement The federal statute passed by Congress The federal treaty The state law

Answer choice B is correct. When a federal statute conflicts with an executive agreement, the federal statute takes precedence over the executive agreement. Here, the federal statute and the executive agreement directly conflict with one another, so the federal statute governs whether the territory can buy bananas from the island nation. Answer choice A is incorrect because, even though the President does have the power to enter into executive agreements with foreign nations, the executive agreement is subject to federal statutes. Answer choice C is incorrect because a federal statute and a federal treaty are roughly equivalent. When the two conflict, whichever was enacted more recently would govern. However, here, the two do not conflict. The federal treaty does not govern whether the territory can buy bananas from the island nation; it only requires that a mark of origin be included on those bananas. Answer choice D is incorrect because federal actions, whether statutes, treaties, or executive agreements, take precedence over state laws.

Question 6594 A state sales tax applied to the sale of most items, including newspapers and magazines. A state statute has recently taken effect that exempts from this tax periodicals that are published or distributed by any religious organization and that consist wholly of writings promulgating the teachings of that religion. The statute requires the publisher of a qualifying periodical to demonstrate to a specified state tax official that it is a religious organization. The statute also specifies that the official may not question the reasonableness of the applicant's religious belief. The publisher of a magazine that does not qualify for this special tax exemption has filed an action in federal court, seeking a declaratory judgment that the exemption statute violates the Establishment Clause of the First Amendment. Should the court find that the tax exemption is constitutional? Answers: No, because a religious organization cannot constitutionally qualify for a tax exemption. No, because the sales tax exemption is confined solely to religious organizations. Yes, because the exemption does not favor one religious sect over another. Yes, because the state, in granting the exemption, does not question the reasonableness of an organization's religious belief.

Answer choice B is correct. When a governmental program shows preference to one religion over another, or to religion over nonreligion, strict scrutiny applies. This sales tax exemption fails under strict scrutiny because it does not appear to be the least restrictive means available to achieve any compelling state interest. Consequently, it violates the Establishment Clause of the First Amendment. Answer choice A is incorrect because the Constitution does not prevent a religious organization from qualifying for a tax exemption if the exemption has a secular purpose and its primary effect is secular. On the contrary, strict scrutiny applies when the government purposely denies a religious entity access to an otherwise available public benefit purely on account of its religious status. Here, however, this sales tax exemption is unconstitutional because it benefits only religious organizations and has the purpose and effect of advancing religion. Answer choice C is incorrect. Although the First Amendment does prohibit the government from favoring one religious sect over another, it also prohibits the government from favoring religious interests over nonreligious interests. Answer choice D is incorrect. Although this statute respects the Free Exercise Clause prohibition on a governmental official ascertaining the reasonableness of a person's religious beliefs, the statute runs afoul of the Establishment Clause because the tax exemption does not have a secular purpose and advances the interests of religious organizations over nonreligious organizations.

Question 1388 Based on a reliable insider tip, a reporter investigated the testing of a corporation's salon products on animals. During the course of her investigation, the reporter sneaked into the corporate headquarters where the reporter discovered and copied many documents proving the management's efforts to conceal the corporation's animal testing from the public. The reporter's newspaper published several of her stories, resulting in a drastic drop in sales of the corporation's product. The corporation filed a lawsuit against both the reporter for trespass and the newspaper for publishing illegally obtained facts. Can the corporation recover damages in its suit? Answers: Yes, the corporation can recover only from the reporter. Yes, the corporation can recover only from the newspaper. Yes, the corporation can recover from either the newspaper or the reporter. No, the media is protected by the First Amendment when gathering and reporting true information concerning matters of public concern.

Answer choice C is correct because the press has no greater First Amendment rights than does the general public. There is no special privilege allowing the press to invade the rights of others. Members of the press are not immune from the application of generally applicable laws, even if the application of such laws has a negative incidental effect on the ability to gather and report the news. The First Amendment shields the media from liability for publishing information that was obtained illegally by a third party as long as the information involves a matter of public concern and the publisher neither obtained it unlawfully nor knows who did. Here, the newspaper likely knew the information was not lawfully obtained and certainly knows who obtained the information. Therefore, the corporation can recover against both the reporter and the newspaper. Answer choice D is incorrect because this protection does not apply where the media or its employee illegally obtains the information. Answer choice A is incorrect because the newspaper would also be liable to the corporation for publishing facts about the corporation's activities when aware of the source of the illegally obtained information. Answer choice B is incorrect because the reporter would also be liable for illegally obtaining the information. No media protection would be granted to her based on the method of obtaining the information.

Question 6597 During a criminal trial in state court, the prosecutor sought and obtained a dismissal of the case. When the prosecutor subsequently attempted to try the defendant again in state court, the trial court ruled that the prosecutor was barred from doing so. The state appealed this decision. The state appellate court, noting that the Double Jeopardy provision in the state constitution was identical to the Double Jeopardy Clause in the Fifth Amendment of the U.S. Constitution, applied only case law of the U.S. Supreme Court interpreting the federal Double Jeopardy Clause in determining the meaning of the identical provision in the state constitution. Based on this federal case law, the state appellate court ruled that the federal Double Jeopardy Clause of the Fifth Amendment as applied to the state via the Fourteenth Amendment prevented the retrial of the defendant and therefore the provision in the state constitution likewise prevented retrial. When the state's highest court declined to hear an appeal of the appellate court decision, the state timely appealed the decision to the U.S. Supreme Court. Can the U.S. Supreme Court hear this appeal? Answers: No, because the state court decision was based on adequate and independent state grounds. No, because the state's highest court did not rule on the matter. Yes, because the state appellate court relied on the U.S. Supreme Court's interpretation of the federal Double Jeopardy Clause in interpreting state law. Yes, because the Supreme Court can hear appeals from all state court judgments.

Answer choice C is correct. A final state-court judgment that rests on adequate and independent state grounds may not be reviewed by the U.S. Supreme Court. The state-law grounds must fully resolve the matter (i.e., be adequate) and must not incorporate a federal standard by reference (i.e., be independent). When it is not clear whether the state court's decision rests on state or federal law, the Supreme Court may hear the case, decide the federal issue, and remand to the state court for resolution of any question of state law. Here, the Double Jeopardy provision in the state constitution was identical to the federal Double Jeopardy Clause in the Fifth Amendment, and the state appellate court applied the U.S. Supreme Court's interpretation of the federal clause in interpreting the state constitutional provision. On these facts, it appears that the state court has relied on a federal standard by reference. Therefore, the state ground was not independent, and the U.S. Supreme Court can hear the appeal. Answer choice A is incorrect because the state appellate court incorporated a federal standard by reference, and therefore its interpretation of the state Double Jeopardy provision was not independent. Answer choice B is incorrect because the U.S. Supreme can exercise jurisdiction over a final judgment of a state court, even a state trial court. The U.S. Supreme Court is not limited to hearing appeals from judgments of the state's highest court. Answer choice D is incorrect because the U.S. Supreme Court cannot hear appeals from final state-court judgments that rest on adequate and independent state grounds.

Question 7393 A state with heavy snowfall each winter sought to prevent injuries and accidents on the road caused by snow falling off of the roofs of large trucks. To further this interest, the state enacted a statute that requires state-employed road maintenance crews to set up random checkpoints at highway on-ramps and state border crossings during the winter months. At these checkpoints, every truck above a certain size is required to stop so the maintenance crews can make sure any snow on its roof is brushed off. The statute has caused significant traffic delays, but the effect on the rate of accidents on highways in the winter has been minimal. An out-of-state company that ships its goods across the state has started losing business because this program has made it nearly impossible to estimate delivery times due to the delays. As a result, the out-of-state company has brought an action challenging the constitutionality of the state statute. Is the company likely to succeed in its constitutional challenge to the state statute? Answers: No, because the state statute does not discriminate against out-of-state commerce. No, because the state statute serves an important state interest. Yes, because the state statute imposes an unreasonable burden on interstate commerce. Yes, because the state statute is not rationally related to a legitimate state interest.

Answer choice C is correct. A state regulation that is not discriminatory may still be struck down as unconstitutional if it imposes an undue burden on interstate commerce. The courts will balance, case by case, the objective and purpose of the state law against the burden on interstate commerce and evaluate whether there are less restrictive alternatives. If the benefits of the state law are grossly outweighed by the burdens on interstate commerce, then even nondiscriminatory regulation may be struck down. Here, the state statute requiring these checkpoints is incredibly burdensome on interstate commerce, and appears to have had only minimal effects on road safety. Therefore, even though it is not facially discriminatory against out-of-state commerce, the statute likely violates the Dormant Commerce Clause. Answer choice A is incorrect because a state regulation that is not discriminatory may still be struck down as unconstitutional if it imposes an undue burden on interstate commerce. Answer choice B is incorrect because, even if this statute serves a state interest, on these facts that interest does not outweigh the burden the statute has imposed on interstate commerce. Answer choice D is incorrect because it improperly applies rational basis scrutiny to a Dormant Commerce Clause violation.

Question 5946 A state statute created a scheme by which the prices paid by its citizens for prescription drugs were subject to state regulation. The Supreme Court found that this statute violated the Dormant Commerce Clause of the U.S. Constitution. Congress subsequently passed legislation, signed into law by the President, that authorized the scheme adopted by the state in the form of a federal statute. The constitutionality of this federal statute has been challenged in federal court. How is the court likely to rule on this challenge? Answers: Uphold the challenge, because of the separation of powers doctrine. Uphold the challenge, because the job of defining the constitutionality of a statute rests with the Supreme Court. Deny the challenge, because of the plenary power granted to Congress under the Commerce Clause of Article I, Section 8. Deny the challenge, because of the Supremacy Clause of Article IV, Section 2.

Answer choice C is correct. Because Congress has exclusive authority over interstate commerce, it may explicitly permit states to act in ways that would otherwise violate the Dormant Commerce Clause. Even assuming that the scheme in question only affected intrastate activity, Congress would have the power to legislate so long as there was a rational basis for concluding that the "total incidence" of the activity in the aggregate substantially affects interstate commerce. Answer choice A is incorrect. Although the separation of powers doctrine has been advanced as support for the Supreme Court's role as the final arbiter of the constitutionality of a federal law, Congress may explicitly permit states to act in ways that would otherwise violate the Dormant Commerce Clause. Answer choice B is incorrect because, although the Supreme Court is generally the final arbiter of the constitutionality of federal as well as state statute, Congress may explicitly permit states to act in ways that would otherwise violate the Dormant Commerce Clause. Answer choice D is incorrect. Although the Supremacy Clause provides that federal law supersedes conflicting state law, in this case the federal statute approves of the scheme adopted by the state statute rather than prohibiting, or otherwise adopting a scheme that conflicts with, the scheme adopted by the state. Therefore, the Supremacy Clause is not determinative of the federal statute's constitutionality.

Question 1350 Congress enacted a statute prohibiting discrimination in the rental of residential property anywhere in the United States on the basis of sexual orientation or preference by any person or entity, public or private. Which of the following provisions provides the strongest basis for Congress's authority to enact this statute? Answers: The enforcement clause of the Fourteenth Amendment. The privileges and immunities clause of Article IV. The commerce clause of Article I, Section 8. The general welfare clause of Article I, Section 8.

Answer choice C is correct. Congress can regulate the rental terms for residential property pursuant to the commerce clause because such rentals constitute economic activity that, in the aggregate, has a substantial effect on interstate commerce. Answer choice A is incorrect because the Fourteenth Amendment's enforcement clause supports only congressional regulation of state action. The enforcement clause therefore would not support application of the statute to private individuals and entities. Answer choice B is incorrect because the privileges and immunities clause of Article IV forbids certain types of discrimination by states against the citizens of other states. It is not a source of congressional power. Answer choice D is incorrect. The General Welfare Clause applies only to taxing and spending legislation passed by Congress.

Question 7442 A manufacturer entered into a contract with the United States Forest Service, an agency of the federal government, to construct a special plane to be used by the Forest Service in fighting forest fires. During the construction of the plane, as the Forest Service made progress payments to the manufacturer, the supplier of materials obtained, under state law, a valid lien upon the plane for the cost of these materials. Shortly before construction was completed, the manufacturer ceased operation and the Forest Service, as permitted by the terms of the contract, took title to the plane. The supplier's otherwise valid lien is now unenforceable against the Forest Service because of the doctrine of sovereign immunity. Has there been an unconstitutional taking of the supplier's property? Answers: No, because the supplier's lien was not an ownership interest in the plane. in personal, not real, property. Yes, because the Forest Service's action effectively eliminated the supplier's lien on the plane. Yes, because the Takings Clause applies to state action through the Fourteenth Amendment.

Answer choice C is correct. One way in which a taking may occur is through the destruction of property or the elimination of a property right. Here, while the supplier continued to possess a valid lien on the plane, the lien was worthless because the supplier could not enforce it against the U.S. Forest Service due to sovereign immunity. Because this action effectively destroyed the value of the supplier's property lien, it constitutes a taking. Answer choice A is incorrect because the Takings Clause is not limited to possessory interests in property, but extends to property rights, such as an easement or a lien. Answer choice B is incorrect because, although the property subject to a governmental taking is typically a real property interest, the Takings Clause applies to an interest in personal property as well. Answer choice D is incorrect. Although the Fifth Amendment's Takings Clause does apply to state action through the Fourteenth Amendment, and the supplier's property interest arose under state law, it was the U.S. Forest Service, a federal government agency, that effected the taking of the supplier's property interest.

Question 7046 The association that regulated all state-licensed realtors organized its members into 20 regions. Four representatives were chosen in each region to join a board to advocate for the rights of all realtors in that area. The board positions were unpaid, and the association established a rule requiring that two of the four chosen representatives must be black. The stated goal of this rule was to address the history of societal discrimination against people of color in the state. In general, there are fewer black realtors in the state than white realtors. If no federal statute applies, is this rule constitutional? Answers: Yes, because the rule is rationally related to a legitimate governmental interest. Yes, because the rule is intended to help remedy past race-based discrimination. No, because the rule only remedies a general societal injustice. No, because the rule is not substantially related to an important governmental interest.

Answer choice C is correct. Programs that discriminate based on race, even if they favor racial or ethnic minorities, are subject to strict scrutiny; and for a governmental affirmative action program based on race to survive, the relevant governmental entity must show more than a history of societal discrimination. The government—whether federal, state, or local—must itself be guilty of specific past discrimination against the group it is seeking to favor, and the remedy must be narrowly tailored to end that discrimination and eliminate its effects. Here, the state board's attempt to remedy general societal racial discrimination, rather than a specific practice of discrimination, does not meet this requirement. Answer choice A is incorrect because it improperly applies the rational basis test to this rule; strict scrutiny is warranted. Answer choice B is incorrect because, to survive, a governmental affirmative action program must remedy specific past discrimination carried out by the government rather than a general societal injustice. Answer choice D is incorrect because it applies the test for intermediate scrutiny where strict scrutiny should apply.

Question 6585 Latino Americans make up approximately 40 percent of the population of a state. They wish to bring a lawsuit challenging a redistricting plan adopted by their state legislature that they contend will prevent them from exercising their voting strength. In addition to the Equal Protection Clause of the Fourteenth Amendment, which of the following constitutional provisions would best support this action? Answers: Thirteenth Amendment Due Process Clause of the Fourteenth Amendment Fifteenth Amendment Twenty-Fourth Amendment

Answer choice C is correct. The Fifteenth Amendment prohibits both the state and federal governments from denying any citizen the right to vote on the basis of race, color, or previous condition of servitude. The courts have interpreted the right to vote to include the right to have that vote meaningfully counted. Therefore, the Fifteenth Amendment will support this action. Answer choice A is incorrect. The Thirteenth Amendment, in addition to abolishing slavery, authorizes Congress to adopt legislation that eliminates the "badges or incidents" of slavery, including racial discrimination. Unlike the Fifteenth Amendment, the Thirteenth Amendment does not provide that specific acts of discrimination are prohibited, but instead enables Congress to adopt legislation rationally related to eliminating racial discrimination. The Thirteenth Amendment is therefore a weak basis for this action. Answer choice B is incorrect. Although the substantive aspect of the Due Process Clause of the Fourteenth Amendment does protect the right to vote, the challenge here is the denial of this right to a particular minority, rather than the denial of this right on another basis. Answer choice D is incorrect because the Twenty-Fourth Amendment is narrowly focused on the prevention of the payment of a fee in order to vote (i.e., a poll tax) in an election for federal office.

Question 6300 The Judiciary Committee of the U.S. House of Representatives initiated impeachment proceedings against a federal district court judge. The President, a lifelong friend of the judge, considered the grounds for impeachment that were being discussed to be politically motivated and without substantial merit. Prior to any hearing on matter by the House committee, the president pardoned the judge. What effect does this pardon have on the impeachment proceedings against the judge? Answers: The proceedings must stop, because the President's power to pardon is plenary. The proceedings must stop, because the President acted in good faith in granting the pardon. The pardon has no effect on the proceedings because a President's power to pardon does not extend to impeachment. The pardon has no effect on the proceedings because a President may not pardon a person until that person has been convicted.

Answer choice C is correct. The President's power to pardon does not extend to impeachment of a federal official. Answer choice A is incorrect. While the President's power to pardon with regard to federal crimes is often referred to as plenary, it does not extend to impeachment of a federal official. Answer choice B is incorrect because there is no good faith restriction imposed by the Constitution on the President's exercise of the power to pardon. Answer choice D is incorrect because, while a President may not pardon a person with respect to future acts, a President may pardon a person at any time after the offense has been committed.

Question 6598 A proposed federal statute would make it a crime to force anyone to engage in prostitution against his or her will anywhere in the United States. Is this proposed statute likely unconstitutional? Answers: Yes, under the Tenth Amendment, because it invades a traditional area of state regulation. Yes, because Congress cannot enact criminal laws. No, because the Thirteenth Amendment gives Congress the power to eliminate involuntary servitude. No, because the Necessary and Proper Clause allows Congress to legislate for the general welfare.

Answer choice C is correct. The Thirteenth Amendment gives Congress the power to adopt legislation rationally related to eliminating involuntary servitude. This power has been broadly interpreted to allow Congress to regulate both private and government action and is the only Amendment that authorizes Congress to regulate purely private conduct. Therefore, the proposed statute would likely be constitutional as it addresses forcing anyone to work against his or her will. Answer choice A is incorrect because the Tenth Amendment reserves to the states only the powers that are not specifically given to the federal government. The power to eliminate involuntary servitude has been specifically delegated to Congress by the Thirteenth Amendment. Answer choice B is incorrect. Although Congress cannot pass bills of attainder (i.e., legislative acts that criminally punishes some person or group without a trial) or enact ex post facto laws (i.e., a retroactive change to a criminal or penal law), there is no general prohibition keeping Congress from enacting criminal laws. Answer choice D is incorrect because the Necessary and Proper Clause gives Congress the power to enact any legislation necessary and proper to execute any authority granted to any branch of the federal government, but this clause is not an independent source of power.

Question 6283 By statute, a state permits gambling by charitable organizations through the use of pull-tab tickets. There is no federal restriction on this type of gambling. The state requires that these tickets be manufactured within the state. This requirement serves the state's legitimate and important interest in ensuring the security and integrity of pull-tab gambling by allowing state regulators to closely monitor the manufacturing processes. In addition, the requirement permits the state to avoid incurring the cost of out-of-state inspection trips. A corporate out-of-state manufacturer of these tickets has challenged the constitutionality of the in-state manufacturing requirement. The manufacturer observes that another state, which also permits gambling through the use of pull-tab tickets, sends its regulators out-of-state and imposes the cost of doing so on the out-of-state manufacturers. Should the court reject this challenge? Answers: Yes, because the requirement serves a legitimate and important state interest. Yes, because there is no federal restriction on this type of gambling. No, because the state requirement violates the Dormant Commerce Clause. No, because the state requirement violates the Comity Clause of Article IV.

Answer choice C is correct. Under the Dormant Commerce Clause, a state may not discriminate against out-of-state commerce. Here the state's requirement that pull-tab tickets be manufactured within the state clearly discriminates against out-of-state commerce since it protects in-state manufacturers from out-of-state competition. Such discriminatory regulation is only upheld if the state or local government can establish that an important local interest is being served and no other nondiscriminatory means are available to achieve that purpose. Because a nondiscriminatory means is available, the regulation cannot be upheld. Answer choice A is incorrect because, even though the state has a legitimate and important interest in avoiding costs related to this form of gambling, the state must establish that no other nondiscriminatory means are available to achieve its purpose. However, the manufacturer has pointed out that another state has adopted a nondiscriminatory means (imposing the cost of out-of-state inspections on the out-of-state manufacturers) for achieving that purpose. Answer choice B is incorrect. Congress has exclusive authority over interstate commerce, so it may explicitly permit states to act in ways that would otherwise violate the Dormant Commerce Clause. Although the federal government permits pull-tab gambling, there is no indication that Congress has expressly allowed or affirmatively contemplated the discrimination in question. Answer choice D is incorrect because the Comity Clause only applies to "citizens," not corporations, and the plaintiff is a corporation.

Question 6584 It has been proposed that the federal statutory exemption for interest earned by the holders of bonds issued by states to finance public works projects, such as highways and educational facilities, be repealed. These bonds are the primary method by which many states finance such projects. Since imposing an income tax on the bond interest would have the effect of increasing the interest that the state must pay on these bonds, this change would have a substantial adverse economic impact on the cost of these projects to the state. Would the repeal of this federal statutory exemption be constitutional? Answers: No, because taxing this income would have a substantial adverse economic effect on the states. No, because the Tenth Amendment prohibits federal taxation of a state. Yes, because the federal government may impose taxes on the states just as the states may impose taxes on the federal government. Yes, because the tax would not be imposed directly on the states.

Answer choice D is correct. A federal tax that is not directly imposed on a state, but instead on payments made by the state to a private person, is constitutional. Consequently, interest payments made by the state to its bondholders may be taxed. Answer choice A is incorrect because a state does not enjoy immunity from a tax that has a substantial adverse economic impact on the state, but is not directly imposed on the state. Answer choice B is incorrect. Although there may be some protection for a state under the Tenth Amendment from federal taxation, there is clearly no immunity when the tax falls on payments made by the state to a private person, such as the payment of interest to the holders of state bonds. Answer choice C is incorrect because the basis for this mutuality argument is false. A state may not impose taxes directly on the federal government.

Question 3245 A state adopted a blanket primary system for choosing elected state officials. Under this system, all candidates participate in a single primary election in which all voters cast ballots. A voter is free to vote for any candidate, and a member of one party may vote for a candidate affiliated with another party. The two candidates who receive the greatest number of votes, regardless of party, advance to the general election. On the ballot, the candidates are identified only by name. Each political party is free to select its nominee by whatever method it chooses. Is this primary system likely to withstand a constitutional challenge? Answers: No, because a state must permit a candidate to designate the candidate's party affiliation on the ballot. No, because a blanket primary system is per se unconstitutional. Yes, because a state is free to design and conduct a primary as it sees fit. Yes, because the primary system doesn't determine a party's nominee.

Answer choice D is correct. A state may institute a blanket primary system in which all voters participate. However, the First Amendment freedom of expression and association prohibit states from interfering with a political party's endorsement of a candidate or selection of a candidate. Because this system does not appear to do either one, it is likely constitutional. Answer choice A is incorrect because a state is likely not required to designate the political party with which a candidate is associated. A state may refuse to list a candidate's party affiliation, despite being chosen as the party's candidate, when the candidate has also been selected by another party as its candidate. Since the refusal did not impose a severe burden on the party's rights, it does not face the strict scrutiny test. Answer choice B is incorrect because a blanket primary is not unconstitutional. Answer choice C is incorrect because while a state is generally free to design and conduct a primary as it sees fit, a state is subject to constitutional restrictions, such as the First Amendment.

Question 5860 Congress enacted a statute permitting states to enact any law regarding the regulation and taxation of persons in the financial services industry, provided that any such law is consistent with federal statutes regulating the industry. A state enacted a statute that taxed all out-of-state financial services corporations doing business in the state. However, the statute did not tax local financial services corporations. A group of out-of-state financial service corporations joined in a court action challenging the constitutionality of the state statute. Assuming the state statute is not inconsistent with any federal statute, is it constitutional? Answers: No, because it violates the Dormant Commerce Clause. No, because it violates the Comity Clause. Yes, because a state may tax a corporation for doing business in the state. Yes, because Congress specifically permitted this type of statute.

Answer choice D is correct. Because Congress has exclusive authority over interstate commerce, it may explicitly permit states to act in ways that would otherwise violate the Dormant Commerce Clause. In this case, Congress, by statute, permitted states to enact any law, including a tax law, regarding the financial services industry that is consistent with applicable federal statutes. Accordingly, the state enacted a statute taxing out-of-state financial services corporations doing business in the state. Although the state statute discriminated against out-of-state corporations, the statute enacted by Congress permitted such discrimination. Answer choice A is incorrect because, as stated above, the discrimination was congressionally permitted. Answer choice B is incorrect because the protections afforded by the Comity Clause do not apply to corporations. Answer choice C is incorrect. While a state may impose a tax on a corporation for doing business in the state, such tax may not discriminate against out-of-state corporations, which this tax clearly does.

Question 5882 A human rights group conducted a study regarding the treatment of federal offenders who are non-U.S. citizens in the public prison system. The group concluded that there was a disturbing pattern of neglect and abuse of non-U.S. citizens in federal prisons. In an effort to improve the treatment of federal offenders who are non-U.S. citizens, Congress enacted a statute that required the Federal Bureau of Prisons to transfer non-U.S. citizens convicted only of immigration offenses to for-profit, private prisons. The statute sets forth guidelines as to how and when the prisoners who are non-U.S. citizens should be transferred to private prisons, as well as criteria regarding which prisoners should be transferred. Is the statute constitutional? Answers: No, because Congress has exceeded its power to delegate. No, because Congress cannot delegate its power to another branch of government. Yes, because there is compelling government interest to protect prisoner rights. Yes, because there is an intelligible principle to guide the Federal Bureau of Prisons.

Answer choice D is correct. Because Congress is vested by Article I with "all legislative powers," it may not delegate that power to any other branch of government. This principle is known as the "nondelegation doctrine." However, delegation of some of Congress's authority to the executive branch has consistently been held constitutional, so long as Congress specifies an "intelligible principle" to guide the delegate. Here, the statute involves law enforcement, which is an executive function, but the constitutional exercise of that function requires that the executive branch act pursuant to congressional authorization as provided by law. Because the statute sets forth guidelines as to how and when non-U.S. citizen prisoners should be transferred to private prisons, as well as criteria regarding which prisoners should be transferred, this legislative delegation passes the "intelligible standards" requirement. Answer choice A is incorrect. There is almost no limitation on the ability of Congress to delegate to the executive and judiciary branches—even broadly phrased standards have been upheld. As noted with regard to answer choice D, this delegation is proper. Answer choice B is incorrect. Law enforcement is an executive function, but the executive branch needs congressional authorization as provided by law to exercise that function. Answer choice C is incorrect. Although the government may have a compelling government interest in making sure that a prisoner's basic human rights are being protected, the central issue here is whether the delegation to the Federal Bureau of Prisons was constitutional. There is no indication that the transfer violates the constitutional rights of the prisoners themselves.

Question 7048 Due to the rising cost of water, Congress enacted a statute that allocated $20 million to fund a consortium of independent non-profit water organizations. The purpose of the consortium was to address the various issues causing a rise in the cost of water and water shortages across the United States, and to fund research into solutions. The statute provided clear guidelines about what the consortium should research and what it could spend, and specified that the remaining funds would be allocated for infrastructure improvements. In addition, the statute delegated to the Department of the Interior the power to select the water organizations to participate in the consortium. Is the statute constitutional? Answers: No, because Congress is delegating its authority to the executive branch of the government. No, because the statute exceeds the scope of Congress's investigative power. Yes, because Congress has the power to enact this legislation under the Necessary and Proper Clause. Yes, because Congress has the power to spend for the general welfare of the public.

Answer choice D is correct. Congress has the power to spend for the general welfare of the public. Here, spending money to figure out ways to cut the cost of water and reduce water shortages is a proper use of Congress's power to spend for the general welfare. Answer choice A is incorrect. Congress may delegate authority to the Department of the Interior so long as Congress specifies intelligible standards for the delegatee to follow. The facts indicate that the statute lays out guidelines for how the funds will be allocated. Therefore, the delegation is not unconstitutional. Answer choice B is incorrect. Congress's investigatory power may extend to any matter within a "legitimate legislative sphere," and funding research and infrastructure improvements for the conservation of water certainly falls within that sphere. Answer choice C is incorrect because the Necessary and Proper Clause is not an independent source of power; it merely gives Congress the power to execute specifically granted powers. Thus, the Necessary and Proper Clause cannot be the only source for Congress's authority to enact this statute.

Question 7052 Pacific Salmon is a valuable fish that shapes the lives of many people and fisheries along the Pacific Northwest, but many stocks are endangered. Congress recently enacted a statute creating a 10-person committee with the authority to research possible methods to preserve these endangered salmon stocks. The statute also gave the committee the authority to issue and enforce rules implementing the preservation methods the committee determined were appropriate. The statute provided that committee members would be appointed by the Fish and Wildlife Agency, an agency of the federal government within the Department of Interior. Is the statute unconstitutional? Answers: No, because Congress gave the Fish and Wildlife Agency, an agency that is within the Department of the Interior, the authority to appoint the members of the committee. No, because the congressional power to legislate for the general welfare permitted Congress to create and delegate rule-making powers to this committee. Yes, because Congress can never delegate rulemaking authority to the executive branch. Yes, because Congress cannot delegate by statute powers that it does not possess

Answer choice D is correct. Congress may not appoint members of a body with administrative or enforcement powers. Such persons are "officers of the United States" and must be appointed by the President. Here, because the fishery committee has enforcement powers, the members must be appointed by the President, not by Congress. Therefore, any attempt by Congress to delegate this appointment authority makes the statute unconstitutional. Answer choice A is incorrect because, as stated above, only the President can appoint the members of this committee, and Congress has no authority to delegate this power to the Department of the Interior. Answer choice B is incorrect because Congress does not have the power to regulate for the general welfare. Congress only has the power to tax or spend for the general welfare under its taking or spending powers. Answer choice C is incorrect because it misstates the law. Delegation of some of Congress's authority to the executive branch has consistently been held constitutional, so long as Congress specifies an "intelligible principle" to guide the delegate. Almost any legislative delegation passes the "intelligible standards" requirement, so even broadly phrased standards have been upheld.

Question 6596 Investors brought an action under federal law for fraud in the sale of securities against an investment company. The action was dismissed with prejudice by the federal district court because it was not timely filed. The investors did not appeal this dismissal. Congress then passed legislation permitting the investors to reinstate this action. The investors petitioned the district court for reinstatement of their action. Of the following, which would serve as the best ground for the investment company to challenge the constitutionality of this law? Answers: Bill of Attainder Clause of Article I, Section 9 of the U.S. Constitution Due Process Clause of the Fourteenth Amendment Ex Post Facto Clause of Article I, Section 9 of the U.S. Constitution Separation of powers doctrine

Answer choice D is correct. Congress may not reinstate the right to bring a legal action after the judgment in the action has become final. Requiring a federal court to do so violates the separation of powers doctrine. Once a judicial decision becomes the final word of the federal judiciary with regard to a particular case or controversy, Congress may not declare by retroactive legislation that the law applicable to that particular case was different from what the courts said it was. Answer choice A is incorrect. A bill of attainder is a legislative act that declares a person or group of persons guilty of some crime and punishes them without a trial. Although the federal government as well as the states are constitutionally prohibited from enacting such legislation, the statute in question is not a bill of attainder because it did not declare the defendant investment company guilty of a crime and punish it without a trial. Answer choice B is incorrect because the Due Process Clause of the Fifth Amendment, not the Fourteenth Amendment, applies to the federal government. Answer choice C is incorrect. Although the constitutional prohibition on ex post facto laws does apply to the federal government as well as the states, the statute in question is not an ex post facto law because it did not constitute a retroactive change in a criminal or penal law (or a civil law with an overriding punitive effect). It merely attempted to permit the reinstatement of the investors' lawsuit.

Question 1356 A protester entered an IRS office during business hours. He denounced the income tax and set fire to pages from his copy of the Internal Revenue Code. The fire was extinguished before it caused any other damage. The protester was arrested and charged with violating a state law that prohibited igniting a fire in a public building. He claimed that his prosecution was unconstitutional under the First Amendment. May the protester constitutionally be convicted? Answers: No, because he was exercising his right to freedom of speech by burning a copy of the code. No, because the copy of the code belonged to him, and thus burning it did not infringe upon a legitimate government interest. Yes, because the burning of the code was conduct rather than speech. Yes, because the state law is narrowly drawn to further a substantial government interest in prohibiting the noncommunicative aspects of the act in question.

Answer choice D is correct. Expressive conduct (or symbolic speech) may be protected as speech, but it is subject to a lesser degree of protection. The protester's burning of the tax code qualifies as expressive conduct protected by the free speech clause of the First Amendment. Government regulation of expressive conduct is upheld if: i) The regulation is within the government's power to enact (e.g., through a local government's police power); ii) The regulation furthers an important government interest; iii) The government interest is unrelated to the suppression of ideas; and iv) The burden on speech is no greater than necessary. Because the state's interest underlying the law that the protester violated (preventing the burning of public buildings) is unrelated to the message communicated by the burning of the tax code, a court will not subject the state law to strict scrutiny. Instead, the court will uphold application of the law to the protester if the law is narrowly tailored to further a substantial government interest, a standard of justification that this law should satisfy easily. Answer choice A is incorrect because the court will be able to meet its burden here, especially as expressive conduct is subject to a lesser degree of protection. Answer choice B is incorrect because the legitimacy (and substantiality) of the government's interest in preventing the burning of public buildings is not undermined by the fact that the protester burned his own copy of the tax code. Answer choice C is incorrect because protected speech may include conduct.

Question 6289 A patient at a government-licensed, private nursing facility was receiving financial assistance from the government for the medical care he received. The status of the patient was reviewed by a committee of physicians working at the facility to determine whether the patient's level of care was appropriate, and whether the patient's continued stay in the facility was justified. The committee determined that the patient no longer needed the level of care provided by the facility, and ordered the transfer of the patient to another nursing facility that offered a lower level of care. Since the patient was receiving financial support through a government-funded program, the committee notified the appropriate governmental official who administered the program. The official in turn contacted the patient and informed him that due to the decision of the committee of physicians, his medical financial assistance would be terminated unless he accepted the transfer. The patient was properly notified of an administrative hearing by the governmental agency who administered the program, and the hearing confirmed the official's decision to terminate his medical financial assistance unless he accepted the transfer. The patient sued the nursing facility for injunctive relief, contending that he was unconstitutionally denied his procedural due process rights with regard to the initial review of his status by the committee of physicians, having received neither notice of the review nor an opportunity to be heard with regard to the review itself. Is the court likely to rule in favor of the patient? Answers: Yes, because the nursing facility was licensed by the government. Yes, because the nursing facility received substantial funding from the government No, because the patient's procedural due process rights were met by notice and the administrative hearing. No, because the decision was made by a committee of physicians working at a private nursing facility.

Answer choice D is correct. In order for a person's procedural due process rights to be violated, the violation must be the result of an action undertaken by the government, not a private person. Although there are circumstances in which an action by a private person is deemed to be governmental action, those circumstances do not exist here. The provision of medical services is not an activity that is traditionally performed exclusively by the government. While private action may also be deemed to be governmental if there is significant government involvement, substantial governmental regulation of a business, such as regulation of a private nursing facility, is not sufficient to constitute significant government involvement. Moreover, the receipt of substantial funding from the government does not transform a private person into a governmental actor, thus the court is not likely to rule in favor of the patient against the nursing facility. Answer choice A is incorrect because the mere licensing or regulation of a private person does not transform an action undertaken by that person into government action. Answer choice B is incorrect because the receipt of substantial funds from the government by a private person does not transform an action undertaken by that person into government action. Answer choice C is incorrect because, while the patient was provided with notice and an administrative hearing with regard to the conditional termination of the financial assistance provided by the government (which satisfied his procedural due process rights with regard to such assistance), the focus of the patient's lawsuit is his right to notice and hearing with regard to the decision made by the committee of physicians, not the related decision regarding his government financial assistance.

Question 4368 In response to the deaths of 15 members of an extended family in a house fire, the state legislature enacted a statute that prohibited more than seven people over age 18 from living together in the same home, unless those people were members of the same immediate family. The legislature enacted the statute based on studies indicating that related individuals are more likely to be injured or killed in fire because, rather than immediately fleeing, they attempt to locate each other before escaping to safety. The study recommended the approach taken by the legislature, but acknowledged that other approaches, such as requiring attendance at fire safety classes, could also be effective. Following the report, members of an extended family of 12 individuals who lived together in the same house filed suit challenging the constitutionality of the statute. Will the court mostly likely conclude that the statute is constitutional? Answers: Yes, because there is no fundamental right affected by the statute. Yes, because it involves a compelling state interest. No, because family members constitute a suspect classification. No, because it is not the least restrictive means for reducing fire deaths of members of an extended family who live together.

Answer choice D is correct. Related persons, including extended family members, have a fundamental right to live together in a single household. Therefore, the strict scrutiny test applies to the statute. Under this test, the law must be the least restrictive means to achieve a compelling governmental interest. Here, the state does have a compelling interest in preventing loss of life due to fire, but there are less restrictive means to achieve that interest, such as requiring members of an extended family to attend fire safety classes, rather than banning such living arrangements altogether. Consequently, the court is likely to conclude that the statute is unconstitutional. Answer choice A is incorrect because related persons, including extended family members, have a fundamental right to live together in a single household. Answer choice B is incorrect because, although the state does have a compelling state interest, it does not achieve it in the least restrictive manner. Answer choice C is incorrect because extended family members do not constitute a suspect classification. Instead, the right of such individuals to live together is a fundamental right.

Question 6286 Three states entered into a compact regarding the rights of each state to use water from a river that flowed through each state. The compact, which was approved by Congress, did not specify an enforcement mechanism. One of the states, contending that the other two states were violating the compact, filed suit in Supreme Court to enforce the compact. Pursuant to the U.S. Constitution, does the Supreme Court have jurisdiction to hear this action? Answers: No, because Congress approved the compact, so the dispute must be resolved by Congress. No, because the Supreme Court's jurisdiction is confined to appellate jurisdiction. Yes, because Congress has provided that the Supreme Court has original jurisdiction over disputes between two or more states. Yes, because the Supreme Court has original jurisdiction over all cases in which a state is a party.

Answer choice D is correct. Section 2 of Article III of the U.S. Constitution specifically provides that the Supreme Court has original jurisdiction over all cases in which a state is a party. Note that Congress cannot expand or limit this jurisdiction, but it can grant concurrent original jurisdiction to lower federal courts. Answer choice A is incorrect because, although an interstate compact may need the consent of Congress, the adjudication of controversies that arise with regard to an interstate compact is the province of the Supreme Court. Answer choice B is incorrect because, while the overwhelming majority of cases heard by the Supreme Court are a consequence of its appellate jurisdiction, the Supreme Court does have limited original jurisdiction pursuant to the Constitution. Answer choice C is incorrect because, while Congress can grant concurrent original jurisdiction to other federal courts, it cannot expand or limit the Supreme Court's original jurisdiction.

Question 1409 An appropriate representative of a resident in a state mental health facility sued an official of the facility in federal court. The resident, who remained a citizen of another state, sought an injunction to compel the official to comply with state law regarding the use of the least restrictive environment approach for the care of the mentally ill. The official moved to dismiss the action as unconstitutional under the Eleventh Amendment. Should the court grant the official's motion? Answers: No, because an injunction, not damages, was sought as a remedy. You Selected: No, because the action was brought against a state official rather than the state. Yes, because a fundamental right is not involved. Correct Answer: Yes, because the action sought the enforcement of state law rather than federal law.

Answer choice D is correct. The Eleventh Amendment prohibits an action in federal court by a citizen of one state against another state when the basis for the action is the violation of state law. Answer choice A is incorrect because, although the plaintiff seeks an injunction rather than damages, the Eleventh Amendment prohibits the action nonetheless, as it is based on state law. Similarly, answer choice B is incorrect because, although the action is brought against the state official rather than the state, the Eleventh Amendment prohibits an action based on state law. Answer choice C is incorrect because, although a plaintiff can pursue an action despite the Eleventh Amendment in order to prevent the enforcement of an unconstitutional state statute, the plaintiff is not advancing such an argument here. Since the plaintiff is not challenging the constitutionality of the state statute under the Due Process Clause of the Fourteenth Amendment, the issue of whether the plaintiff is asserting a fundamental right is irrelevant.

Question 6301 A state statute provides that local county governments within the state may adopt an ordinance that prohibits the sale of alcoholic beverages within the county. A local county government, in accord with this statute, adopted a resolution banning the sale of alcoholic beverages within the county with an alcoholic content in excess of 10 percent. Which of the following provides the best support for the constitutionality of this ordinance? Answers: The Dormant Commerce Clause of the Article I, Sec. 8, of the U.S. Constitution The Tenth Amendment to the U.S. Constitution The Equal Protection Clause of the Fourteenth Amendment to the U.S. Constitution The Twenty-First Amendment to the U.S. Constitution

Answer choice D is correct. The Twenty-First Amendment, in addition to repealing prohibition, specifically gives states the authority to prohibit the transportation or importation of alcoholic beverages into the state for delivery or use within the state. Answer choice A is incorrect because the Dormant Commerce Clause, rather than supporting a state's regulation of alcoholic beverages, serves as a check on such regulation. Answer choice B is incorrect. The Tenth Amendment generally reserves to the states the powers not delegated to the federal government by the Constitution or prohibited to the states. However, the Twenty-First Amendment specifically grants to the states the power to prohibit the transportation or importation of alcoholic beverages in the state for delivery or use within the state. Answer choice C is incorrect because the Equal Protection Clause, rather than supporting a state's regulation of alcoholic beverages, serves as a check on such regulation.

Question 4364 To address a shortfall in the judiciary's budget and to comply with a provision in the state constitution requiring a balanced budget, a state increased, by statute, various court fees, including the fee to file an appeal, and eliminated any waiver of these fees except for a defendant convicted of a capital crime and sentenced to death. A defendant imprisoned after his conviction of a non-capital offense filed an appeal and a request for a waiver of the fee because of his inability to pay. Citing the statute, the state appellate court ruled that the defendant was not entitled to the waiver, as he was not convicted of a capital offense and sentenced to death. The defendant properly challenged this ruling in federal court. Should the federal court uphold the state appellate court's ruling? Answers: Yes, because the fee is rationally related to the goal of adequately funding the state's judicial system. Yes, because state law required payment of the fee. You Selected: No, because a state may not prohibit a prisoner from exercising a fundamental right. No, because the defendant was indigent.

Answer choice D is correct. While most state actions that discriminate against the poor are subject only to rational basis scrutiny, the availability of appeal in a criminal case cannot hinge on ability to pay a filing fee. To deny an indigent the right to appeal through the imposition of a fee violates the Due Process and Equal Protection Clauses of the Fourteenth Amendment. Answer choice A is incorrect because, although there is a rational basis for the state's increase in the various court fees, an indigent accused has a constitutional right to waiver of a fee that prevents him from enjoying a fundamental right, such as access to the courts. Answer choice B is incorrect because, even though the state court decision was based on state law, the state court cannot enforce a state law that conflicts with the federal constitution. Answer choice C is incorrect because, while a prisoner has a right of reasonable access to the court, the law in question did not deny the defendant such access on the basis of his status as a prisoner, but due to his failure to pay the necessary filing fee.


संबंधित स्टडी सेट्स